Instructor’s Guide for ICD-9-CM Diagnostic Coding and ...



Basic Current Procedural Terminology and HCPCS Coding 2016 Edition Instructor’s Manual Notes to Instructors This instructor manual includes resources for instructors. This item and PowerPoint slides are available at . Chapter lesson plans include the following elements: Bullet list summary of lesson focus Objectives Suggested student activities and assignments Key points for lecture notes Websites for faculty or students Additional exercises with answer keys provided to faculty. Exercises include: practice coding operative reports, CPT Assistant exercises, and test bank items Answer key to appendix C is provided (answers not provided to students) The websites listed in this book were current and valid as of the date of publication. However, website addresses and the information on them may change or disappear at any time and for any number of reasons. The user is encouraged to perform general web searches to locate any site addresses listed here that are no longer valid. For an introduction, faculty may consider demonstrating to the student useful sites, such as: Online Communities (coding) AHIMA HIM Body of KnowledgeContents Lesson Plans (including Objectives, Suggested Student Activities and Assignments, Key Points for Lecture Notes, and Test Bank Questions)Chapter 14 Chapter 29 Chapter 315 Chapter 420 Chapters 5, 6, 8, and 933 Chapter 740 Chapter 10 46 Chapter 1150 Full Answer Key to Textbook 201658Answers to Additional Practice Exercises, Appendix B—201692 Chapter 1 Introduction to CPT and HCPCS This chapter will focus on the following topics: Introduction to CPT and HCPCS HCPCS Coding Claim forms (CMS-1500 and CMS-1450) Objectives After completing this chapter, the student should be able to do the following: 1.Define coding 2.Discuss the purpose and future of coding 3.Identify the purpose and uses of CPT 4.State the official publication for CPT coding 5.Define and list the components of HCPCS 6.Distinguish between CPT and National Codes 7.Describe the general principles of medical record documentation 8.Given a list of services, identify what cannot be coded with CPT 9.Differentiate between the uses of CPT, ICD-9-CM, ICD-10-CM, and ICD-10-PCS10.Identify the ICD-9-CM and HCPCS coding assignment necessary for physician services, hospital inpatients, and hospital ambulatory patients 11.Differentiate between CMS-1500 and UB-04 forms 12.Define key words 13.Identify abbreviations Suggested Student Activities and Assignments 1.Read current articles about current trends in coding (for example, Computer Assisted Coding). Write a reaction paper or summary of the article. 2.Go to the CMS website to review resources (for example, HCPCS online file of codes, Medicare Coverage Determination).3.Review the AMA website for updates for CPT Category II and III codes.4.During one of the first lectures, walk the students through the entire coding process and include software tools for coders. Suggestion for FacultyRefer to AHIMA CodeWrite and Webinars to supplement lectures with advanced content concepts. Subscribe to AHIMA CodeWrite here: . Subscribe to AHIMA Webinars here: . Introduce students to web resources. Many of the websites offer videos of surgical procedures that enhance student learning.Key Points for Lecture Notes 1.The American Medical Association (AMA) and some third-party payers developed CPT in the 1960s. It was used as a communication tool between third-party payers and physicians. Unlike ICD-9-CM (and ICD-10-CM), CPT was developed for reimbursement purposes.2.Point out the differences that ICD-10-CM replaced ICD-9-CM in October 2015. Differentiate between ICD-10-PCS and CPT.3.It was not until the mid-1980s that hospitals began to use CPT. The Omnibus Budget Reconciliation Act demanded that hospitals use CPT to report outpatient services. 4.In the textbook, you will find examples of both UB-04 and CMS-1500 forms. Explain that claim forms are electronically submitted. CMS-1500 forms are used only in physician offices. UB-04 is the billing form for all hospital services: inpatient, outpatient, emergency, clinic, and so on. Introduce the HIPAA 5010 electronic transaction standards and how it paves the way for ICD-10-CM/PCS. 5.Some of the routine services in the hospital are contained in a computerized file called a chargemaster. For example, consider a patient seen as an outpatient for a chest X-ray. Almost all facilities have the CPT codes for chest X-ray in the computer file; therefore, coders rarely assign these codes. 6.HCPCS is the most confusing part of the unit. When CMS decided to use CPT for reimbursement to hospitals and physician offices, there was one big hurdle: not everything that is reimbursed by Medicare or Medicaid is contained in CPT (for example: drugs, supplies, and ambulance services). Keep in mind that CMS does not own CPT; therefore, CMS cannot control the contents. To supplement CPT, CMS wrote its own book of codes called Level II National Codes. When billing Medicare for an ace bandage given to a patient, the coder would have to reference Level II codes, because there are no codes in CPT for supplies. 7.Provide a list of services not in CPT. 8.CMS subcontracts with local carriers (now called Medicare Administrative Contractors [MACs]) to manage claims processing in the geographic area. Prior to 2004, CMS gave local carriers the authority to also create codes, if the need arose. These codes were called Level III or Local Codes. The codes were communicated through newsletters called Medicare Bulletins. This two-part system is called HCPCS. Sometimes CPT and HCPCS are used interchangeably but that is not technically correct. 9.Call attention to the fact that CPT and HCPCS is used to identify procedure codes, and ICD-9-CM continues to be the coding system for all healthcare providers for diagnosis codes. ICD-9-CM describes “why” services were performed and CPT and HCPCS describe “what” services were performed. ICD-9-CM is the only system for inpatients (both for diagnoses and procedures) and will continue with implementation of ICD-10-CM/PCS in October 2015. Although third-party payers want CPT and HCPCS for billing purposes on outpatients, some hospitals continue to assign ICD-9-CM procedure codes for their hospitals’ database and for statistical reasons. 10.Many private insurers also accept HCPCS Level II codes. Chapter 1 Test Bank Items Instructions: Choose the best answer for each of the following questions. 1.For the December 7, 2015 patient encounter, the hospital will submit codes on what billing form? A.UB-04B.HCFA-1500C.CMS-1500 D.UHDDS 2.A patient visits a physician’s office for back pain. The services for this patient would be submitted on what claim form? A.HCPCS B.UB-04 C.CMS-1500 D.CMS-1450 3.A patient is seen in a clinic for a laceration of the elbow. The wound required suturing. On the claim form, which of the following types of codes would be assigned to represent the laceration? A.ICD-10-CMB.CPT C.National Codes D.Level III Codes 4.Which of the following code sets captures inpatient procedure codes?A.CPT-4B .HCPCSC.ICD-10-PCSD.ICD-9-CM, Volume 35.HCPCS was developed by: A.American Medical Association B.Department of Health and Human Services C.Centers for Medicare and Medicaid Services D.American Hospital Association 6.What codes will the hospital use on its billing form to present the diagnosis of “fractured humerus”? A.CPT B.HCPCS C.ICD-10-CM D.DSM-IV 7.Which of the following procedures or services could not be assigned a code with CPT? A.Intravascular UltrasoundB.AnesthesiaC.Wound warming deviceD.Glucose tolerance test 8.Which of the following groups and organizations is responsible for maintaining the HCPCS Level II codes? A.American Medical Association B.American Hospital Association C.Centers for Medicare and Medicaid Services D.Local Fiscal Intermediary 9.The CPT book is updated: A.As needed B.For use in January C.Monthly D.For use in October 10.Which of the following can be identified as a National Code? A.J0207B.89840 C.1061F D.0345T 11.The CPT manual is published and maintained by the: A.American Medical Association B.Department of Health and Human Services C.Centers for Medicare and Medicaid Services D.American Hospital Association Chapter 2 Procedural Terminology in Current Use This chapter will focus on the following topics: Format and organization of CPT Conventions and characteristics of CPT Alphabetic Index Abstracting documentation Coding references Objectives After completing this chapter, the student should be able to do the following: 1.Specify the conditions that must be met before a procedure or service is included in the CPT manual 2.Describe the contents of CPT: sections, subsections, subcategory heading, procedure, appendices, and index 3.Interpret conventions and characteristics of CPT: a.Semicolon b.Bullet c.Triangle d.Facing triangles e.Plus sign f.Null symbol g.Circled bullet h.Pending FDA approval symbol i.Resequenced codesj.Reinstated code 4.Describe Category I, II, and III CPT Codes 5.Successfully apply the general rules and guidelines for coding assignment 6.Define key concepts: a.Unlisted procedures b.“See” c.Stand alone 7.Given an operative report, successfully abstract pertinent clinical information 8.Reference official coding guidelines (CPT Assistant) to support accurate coding assignment Suggested Student Activities and Assignments1.Allow the students to work in groups and interpret documentation in the operative report; ask students to identify the main procedure. 2.Administer a self-assessment medical terminology quiz. 3.Look up and discuss unfamiliar terminology from operative reports. 4.Discuss the importance of using reference materials such as anatomical diagrams, surgical descriptions, and CPT Assistant to support accurate coding assignments. 5.Research coding advice from CPT Assistant. 6.Access the AMA website and discuss the uses of Category II and Category III codes. Discuss articles from the AHIMA Body of Knowledge, for example:“Where to Find Answers to Coding Questions,” Journal of AHIMA (Coding Notes), April 2, 2010“Internet Resources for Accurate Coding and Reimbursement Practices,” AHIMA Practice Brief, March 31, 2010“Anatomy of a Physician Coder: Small, medium and large physician practices all feature these multi-tasking, multi-responsibility coders,” Journal of AHIMA, November 2, 2012.Key Points for Lecture Notes The textbook provides a good guide for the lecture in this unit. 1.Semicolon. This is one of the most confusing format concepts. If a code description contains a semicolon and there is one or more code description indented underneath, then the description before the semicolon is a “home description,” and the indented code needs the “home description” to fully complete the code description. Note the second example: Osteotomy; tibia. Code 27705 should read: Osteotomy; tibia. Code 27707 should read: Osteotomy, fibula. The “home description” before the semicolon completes the description. Look at another example: 30150: 30150 Rhinectomy; partial 30160 total If the physician documented that the patient had a total rhinectomy, the correct code would be 30160. Ask the students if they would assign CPT codes 30150 and 30160 together. Answer: no, it does not make sense. It is either partial or total, but not both. This is a good time to introduce students to the term “mutually exclusive.” 2.Category II and Category III Codes: AMA continues to expand the use of codes. 3.Alphabetic Index. The following are tips that are not in the textbook: a.The index is more “free form” than ICD-9-CM. In ICD-9-CM, the index gives you reliable choices, but not always in CPT. The tabular codes rule the coding process. b.After locating the code choice(s), ask the students to make a list to check against the tabular. These codes are tentative and need to be explored fully. Now ask the students to use process of elimination to choose the correct code. Students may have to use reference materials to help support their answer. c.Read, read, and read again. Sometimes two codes look the same, so try and find out the difference. Encourage students to look up unfamiliar medical terms or abbreviations. 4.Coding operative reports. The textbook provides help for abstracting documentation from operative reports for coding. One of the most common mistakes for new coders is to code everything. Emphasize to student that they should not automatically assign codes for approaches and closures. Scan the operative reports for “action” words. Typical action words would be: excision, incision, endoscopy, exploration, and so on. 5.Coding References. Emphasize the importance of using this reference to support their answers. To stress the importance of using references, two questions in the test bank items require students to reference a medical dictionary (questions 10 and 11). 6.AHIMA Coding Community and Body of Knowledge. Ask students to explore the CoPs and Body of Knowledge on the AHIMA website. CodeWrite Community Newsletter is published monthly. Chapter 2 Test Bank Items 1.Reference codes 11200–11201 for removal of skin tags. What is the correct code(s) for removal of 16 skin tags? a.11200, 11200 b.11200 x 16 c.11200, 11201 d.11201 x 16 2. Which of the following can be identified as a CPT code number from the Medicine section? a.97035b.28270 c.88304 d.926113.Reference 11920–19222 for tattooing. What is the correct code(s) for tattooing of 40 sq cm of skin? a.11920, 11921, 11922 b.11921, 11922 c.11922, 11922 d.11920, 11922, 11922 4.Which of the following contains a comprehensive summary of CPT additions, deletions, and revisions since last year? a.Appendix A b.Appendix B c.Appendix D d.Appendix F 5.Reference codes 31360–31368 for laryngectomy. What is the correct code assignment for a laryngectomy with subtotal supraglottic and radical neck dissection?a.31360, 31368b.31368c.31367d.313606.Which of the following can be identified as a new code in CPT 2016? a.27049b.47540c.35500 d.475257.Which of the following contains a complete description of CPT modifiers? a.Appendix A b.Appendix B c.Appendix C d.Appendix D 8.Reference codes 49491–49525 for inguinal hernia repair. What is the correct code for an initial inguinal herniorrhaphy for incarcerated hernia (patient is 47 years old)? a.49496 b.49501 c.49507 d.49521 9.A surgeon performed a procedure that is unfamiliar to the coder and the coder is having trouble locating an appropriate CPT code. What should the coder’s next action be? a.Assign an “unlisted procedure” code b.Call the insurance company c.Research the description about the procedure d.Call the surgeon’s office 10.What is the correct code assignment for: destruction of 2 groups of internal hemorrhoids with use of infrared coagulation? a.46250b.46930 c.46260d.46946 CPT Assistant Exercises Directions: Ask the students to answer these questions after referencing CPT Assistant. True or FalseA surgeon used a trocar technique (placement of a suprapubic tube) to perform cryosurgical ablation of the prostate. CPT code 55873 is the only code needed to report this procedure.TrueFeedback: Placement of a suprapubic tube (trocar technique), as well as ultrasonic guidance and monitoring is included in the work of the procedure. (CPT Assistant, September 2015, page 12). True or FalseA physician performed two trigger point injections in two different muscles. CPT code 20552 would be reported twice. FalseFeedback: Codes 20552–20553 are reported one time per session, regardless of the number of injections or muscles injected (CPT Assistant, May 2003, page 19).True or FalseThe surgeon performs a laparoscopic appendectomy at the same time as the laparoscopic colostomy. Both the laparoscopic, appendectomy (code 44970) and the laparoscopic colostomy (code 44188) codes are reported.False Feedback: According to the parenthetical notes that follow code 44188, it is not appropriate to report 44188 in conjunction with 44970. The appendectomy would be considered incidental to the colectomy (CPT Assistant, April 2006, page 20).True or FalseThe surgeon performed a laparoscopic assisted vaginal hysterectomy using the daVinci? robotic assistance system. This procedure is reported as an unlisted code.FalseFeedback: Currently, the robotic surgical procedures may be reporting using the existing CPT codes based on the procedure performed and the surgical approach used. In this case, the selection of the CPT code would be 58571 or 58573 based on the site of the uterus (CPT Assistant, August 2012, page 13).4.True or FalseWhen reporting a code for a wound repair, the code selection is based on the final size of the wound after it was repaired not on the initial size.TrueFeedback: Reporting should be based upon the final size of the wound (CPT Assistant, October 2014, Page 14). Chapter 3 Modifiers This chapter will focus on the following topics: Introduction of CPT and HCPCS modifiers List of modifiers approved for physician use and hospital outpatient use Appropriate use of CPT and HCPCS modifiers Objectives After completing this chapter, the student should be able to do the following: 1.Identify the purpose of modifiers 2.State the uses of modifiers for surgical procedures 3.Differentiate between the modifiers identified for hospital outpatient use and the complete list of CPT modifiers 4.Differentiate between modifiers 73 and 74 5.Identify the proper use of modifier 59 6.Research Medicare Transmittals and CPT Assistant pertaining to use of modifiers 7.Given a scenario, append a CPT code with the correct modifier Suggested Student Activities and Assignments Search the CMS website for Medicare Transmittals pertaining to use of modifiers. Listen to AHIMA Audio Seminars.Discuss articles from the AHIMA Body of KnowledgeKey Points for Lecture Notes 1.Emphasize the difference between modifiers used for hospital outpatients vs. modifiers for physicians. CPT code books have a quick reference on the reverse side of the front cover page. Mention that the dash between the code and modifier is NOT part of the code assignment but used only for ease in reading the written code assignments. 2.Students should answer the following questions to determine if a modifier should be appended to the CPT code: Will a modifier add more information regarding the anatomic site (for example, LT, T5)? Will a modifier help to eliminate the appearance of duplicate billing, or the appearance of unbundling? 3.Although the National Correct Coding Initiative (CCI) is discussed in Chapter 4, faculty should briefly introduce the concept during the discussion on modifiers. CPT included a Colonoscopy Decision Tree in the professional edition. The decision tree highlights the difference between diagnostic and therapeutic, and guides the coder to the correct assignment if the complete endoscopic procedure is not performed.Chapter 3 Test Bank Items For each of the following, append CPT or HCPCS modifiers to the following CPT procedure codes. 1.What HCPCS Level II modifier would be appended to a laboratory test that was ordered by the court system?GUGZH9HA2.True or FalseThe correct code assignment for an arthrocentesis, ring finger of left hand is 20600–LT.False Feedback: 20600–F3 (Note: Remind students that the F and T modifiers are for fingers and toes, not metatarsals and metacarpals) 3.True or FalseThe correct code assignment for a closed reduction of fractured phalange, 5th digit, right foot is 28515–T9.True 4.True or False Bilateral maxillary sinusotomies is reported as 31020.False Feedback: Append modifier 50 to the CPT code.5.Which of the following modifiers would be appended to a CPT code for repair of the right upper eyelid?a.E1b.E2c.E3d.E4 6.True or FalseThe correct code assignment for an extracapsular cataract extraction with insertion of lens, OS is 66984–LT. True 7.True or FalseCode 55250–50 is reported for a bilateral vasectomy. FalseFeedback: No modifier is needed, code description states “unilateral or bilateral”.8.True or FalseThe patient reports that her breasts are too large and as a result, she experiences severe back and shoulder pain. The physician performs a reduction mammoplasty and the correct code assignment is 19318–50. True 9.True or FalseIncision and drainage of carbuncle on left hip is performed and the correct code assignment is 10060–LT. FalseFeedback: No modifier is appended because the CPT description does not specific site and the procedure was performed on the skin.10.Operative Report Preoperative Diagnosis: Cholecystitis with cholelithiasis Postoperative Diagnosis: Same Operative Procedure: Laparoscopic cholecystectomy Indications: A 77-year-old woman experiences upper abdominal pain and has been diagnosed with cholelithiasis. The risks and benefits of the procedure have been explained in detail. Technique: With the patient under general anesthesia, the abdomen was prepped and draped in the usual fashion. A small infraumbilical skin incision was made, carried down through the adipose tissue. The fascia was opened in the midline, and the peritoneal cavity under direct vision using open laparoscopic technique. There was adequate insufflation of CO2. A 10-mm trocar was introduced into the upper abdomen to the right of the midline, two 55-mm trocars in the right upper quadrant area under directed camera vision. Examination noted that there were multiple adhesions in the gallbladder area. At this point, I was notified that the patient’s blood pressure was 150/80 and then dropped to 90/55. The blood pressure was stabilized but the decision was to abort the procedure at this time. All trocars were taken out under direct camera vision. The CO2 was desufflated. Infraumbilical incision was closed using 4-0 Vicryl subcuticular sutures, and Steri-Strips. She will be closely monitored and I will contact her primary care physician to discuss her condition. For hospital outpatient reporting, what is the correct code assignment? 47562–5247562–5347562–7347562–74CPT Assistant Exercises Directions: Ask the students to answer these questions after referencing CPT Assistant. True or FalseThe surgeon performed a screening colonoscopy and reached the cecum, but due to a poor prep, the procedure was discontinued and rescheduled. Modifier 53 would be appended to the colonoscopy code for the physician’s service.TrueFeedback: It is appropriate to append the modifier. (CPT Assistant, February 2014, page 11)True or FalseModifier 59 may be appended to an unlisted code such as 29999. False Feedback: It is not appropriate to append any modifier to an unlisted code because modifiers provide the means by which the reporting physician can indicate that a service or procedure has been altered by some specific circumstance, but not changed in its definition or code. Unlisted codes do not describe a specific service; therefore, it is not necessary to utilize modifiers (CPT Assistant, September 2005, page 9).3.True or FalseA physician states that an acoustic reflex test of the left ear was performed. CPT code 92568–LT would be reported. FalseFeedback: As stated in the CPT guidelines for audiologic function tests, all descriptors refer to testing of both ears. Use modifier 52 if the test is applied to one ear instead of two (CPT Assistant, June 2004, page 10). 4.True or FalseCPT code 69610 (tympanic membrane repair) is considered to be unilateral. TrueFeedback: Unilateral. If the procedure is performed bilaterally, modifier 50, Bilateral procedure, should be appended (CPT Assistant, March 2003, page 21).5.True or False If everything listed in code 95922 is not performed, the code is reported with modifier 52. TrueFeedback: CPT code 95922 requires both a passive title and a Valsalva maneuver be performed. If only one or the other is performed, then modifier 52, Reduced services, should be appended to the code (CPT Assistant, June 2003, page 11).Chapter 4 Surgery This chapter will focus on the following topics: Surgical package Separate procedure National Correct Coding Initiative (NCCI) Modifier usage Surgical coding Objectives 1.Surgery Overview a.Describe the organization of the surgical section in CPT b.List components of a surgical package c.Distinguish between the CPT definition of surgical package and Medicare definition d.Define and cite examples of “separate procedure” e.Define National Correct Coding Initiative (NCCI) f.Differentiate between modifiers for physician use versus modifiers for hospital outpatient use g.Given a procedure or operative report, correctly assign CPT codes 2.Integumentary System Subsection a.Identify guidelines for coding excision of lesions b.Differentiate between benign and malignant lesionsc.Distinguish between excision and destruction of lesions d.Differentiate between simple, intermediate, and complex wound repairs e.Successfully apply guidelines for coding wound repairs f.Differentiate between the types of skin grafts g.Differentiate between excisional and incisional breast biopsies h.Define lesion, epidermis, dermis, subcutaneous tissue, benign, and malignant 3.Musculoskeletal System Subsection a.Identify the appropriate use of casting/strapping codes b.Differentiate between surgical and diagnostic endoscopies 4.Respiratory System Subsection a.Identify guidelines for coding nasal endoscopy procedures b.Differentiate between direct and indirect laryngoscopies c.Define bronchoscopy d.Describe procedures commonly performed with bronchoscopies 5.Cardiovascular System Subsection a.Differentiate between injection proceduresb.Define CABG and explain the guidelines for coding assignment c.Describe the documentation necessary to accurately assign codes for pacemaker procedures d.Identify the components of coding Interventional Radiology e.Define an AV fistula and indications for the procedure f.Differentiate between the coding selections for AV fistulasg.Differentiate between the venous access devices 6.Digestive System Subsection a.Define ERCPb.Define proctosigmoidoscopy, sigmoidoscopy, and colonoscopy c.Describe the methods for removal of tumors/polyps during a colonoscopy procedure d.Explain the coding guidelines for incomplete colonoscopy procedures e.State guidelines for coding biopsies with lesion removals during GI endoscopyf.Differentiate between the surgical procedures codes for treatment of hemorrhoids g.Define key terms associated with type, presentation, and/or location of hernias 7.Urinary System Subsection a.Define the various urodynamic procedures b.Describe the organization of the urinary system subsection 8.Male Genital System Subsection a.Differentiate between the removals of lesions described in Male Genital Subsection versus those described in the Integumentary System subsection 9.Laparoscopy/Hysteroscopy Subsection a.Differentiate between laparoscopic and hysteroscopic procedures 10.Nervous System Subsection a.Differentiate between the spinal injection procedures 11.Eye and Ocular Adnexa Subsection a.Differentiate between extracapsular and intracapsular cataract extraction b.Identify the procedures included in the codes for cataract extractions 12.Auditory System Subsection a.Differentiate between tympanostomy and myringotomy for insertion of ventilating tubes. Suggested Student Activities and Assignments 1.For each system, provide a list of applicable medical terms to review with students. For example, in the Integumentary System, have students search the web for definitions and images to differentiate between the types of skin lesions.2.Assign students a surgical procedure to research and present to the class. 3.Identify web references that describe surgical procedures. Note that many professional associations have websites that provide health information. For example, the American Academy of Otolaryngology: library references for surgical procedures. 5.Show videos of various surgical procedures. Interactive tutorials can be found on the National Library of Medicine’s website (Medline Plus). In addition, the website called Spine-Health has excellent videos to explain spinal fusion. Key Points for Lecture Notes 1.Encourage students to carefully read any “Notes” that appear in the CPT book. 2.Explain the most common procedures so the students can identify the purpose and technique utilized. 3.Concentrate on key terms. 4.Remind students to not code what they don’t understand. 5.Supplement lectures with anatomic pictures and videos (for example, Medline Plus).6.When choosing CPT codes, ask students to explain the difference between each of the codes under consideration Websites for Faculty and Students It is important that students build a library of resources to research surgical procedures (see web resources in appendix A. Medline Plus provides real surgical videos and interactive tutorials.Chapter 4 Test Bank Items Part I: Multiple Choice 1.The patient had a laparoscopic incisional herniorrhaphy for a recurrent reducible hernia. The repair included insertion of mesh. What is the correct code assignment?a.49565 b.49565, 49568 c.49656 d.49560, 495682. In the clinic, the physician performed a simple incision and drainage of a pilonidal cyst.a.10060b.10061c.10080d.100813.The patient had a total abdominal hysterectomy with bilateral salpingectomy. The coder selected the following codes 58150 and 58700. The assignment of these two codes together would be referred to as: a.Global packing b.Unbundling c.Maximizing d.Optimization 4.An asymmetric nevi, total excision size of 1.0 cm x 2.0 cm was removed from the patient’s back. Pathology report identifies the specimen as “interdermal nevi.” What is the correct CPT code assignment for this procedure? a.11603 b.11402c.11602d.11403 5.The surgeon performed an open mitral valve replacement with cardiopulmonary bypass.334053342233465334306.A cystourethroscopy with ureteroscopy was performed to remove a calculus lodged in the left ureter.523105232052330523527. Which of the following terms is associated with graft material harvested from an animal to be used for a human?AllograftXenograftAutograftZenograft8.EGD (transoral) with removal of a piece of a chicken bone. a.43194b.43247 c.43215d.432359.What is the correct code assignment for a cervical conization with loop electrical excision?5746057461575105752210.Which of the following procedures can be identified as destruction of lesions? a.Shaving of epidermal lesion b.Cryosurgery of lesion c.Excising skin tagsd.Paring of hyperkeratotic lesions 11.What is the correct CPT code assignment for electrosurgical removal of three (3) nevi of the arm (size approximately 2.0 cm, 1.5 cm, 0.5 cm)? a.11400, 11402, 11402 b.11056 c.11200 d.17000, 17003, 17003 12. The surgeon replaces the peripherally inserted central venous catheter (PICC) through same access.a.36578b.36580c.36584 d.3658513.What is the correct CPT code assignment for hysteroscopy with lysis of intrauterine adhesions? a.58555, 58559 b.58559 c.58559, 58740 d.58555, 58559, 58740 14.The physician documents that she changed the cardiac pacemaker battery. In CPT, the battery is called a(n): a.Generator b.Electrode c.Dual system d.Cardioverter 15.The surgeon performs an open thrombectomy of an AV fistula, without revision of the dialysis graft. What is the correct CPT code assignment for this procedure?a.36870b.36833c.36831d.3683216.A patient is seen in the emergency department following an accident. The physician documents that the wound required multiple layers and extensive undermining. According to CPT definitions, this type of repair would be classified as: a.Simple b.Intermediate plex d.Advancement flap 17.The physician performed a colonoscopy that extended to the cecum and used a snare to remove a polyp of the transverse colon.4533845378, 45388453854538818.The physician performs an exploratory laparotomy with bilateral salpingo-oophorectomy. What is the correct CPT code assignment for this procedure? a.49000, 58940, 58700 b.58940, 58720–50 c.49000, 58720 d.58720 19.Needle aspiration of a cyst of the left breast.a.19125–LTb.19000–LT c.19260–LTd.19100–LT20.Shaving of 1.5 cm epidermal lesion, scalp.a.11422b.11622c.11302d.1130721.Closed reduction of right radial shaft fracture.a.25500–RTb.25505–RTc.25520–RTd.25565–RT 22.The 25-year-old patient had an initial repair of incarcerated incisional hernia with insertion of mesh to support the abdominal wall a.49507b.49521, 49568c.49561d.49561, 49568 23.A patient was taken to the endoscopy suite. The flexible endoscope was passed from the mouth into the esophagus and continued into the stomach and into the duodenal bulb. Based on this documentation, what CPT code would be selected to represent this procedure? a.43200 b.43234 c.43235 d.43260 24.A physician documented the following surgical procedure for treatment of chronic otitis media: “Myringotomy with insertion of ventilating tubes in both ears. Performed under general anesthesia.” What is the correct CPT code assignment for this procedure? a.69421–50 b.69421–LT, 69421–RT c.69436–50 d.69433–LT, 69433–RT 25.The surgeon created a twist drill hole for evacuation of a subdural hematoma.a.61105b.61107c.61108d.61154Part II: Operative Reports 26.Operative Report Preoperative Diagnosis: Internal derangement left knee Postoperative Diagnosis: Tear of lateral meniscus Operative Procedure: Left knee arthroscopy, partial meniscectomy The arthroscope was inserted through the routine superolateral portal as well as an inferomedial portal for insertion of scope and instruments. The knee joint was then examined in routine manner, the medial meniscus was intact. The lateral meniscus was partially detached and this portion was removed. No other defects were noted. The knee was irrigated well using normal saline. The instruments were removed from the knee. Wound closed with #4-0 nylon and dressed. Estimated blood loss 0. Intravenous fluids 1000 cc. Specimen: meniscus. Complications: 29880–LT29881–LT 29882–LT29883–LT27.Operative Report Preoperative Diagnosis: Mass, superior aspect of the left breast Postoperative Diagnosis: Benign mass, superior aspect of the left breast Operation: Excision The patient is a female who has had a lump palpable over the superior aspect of the left breast for the past several months. It has been observed in the office. I had done a needle aspiration and did not get any fluid out. After multiple observations, the patient was very concerned about carcinoma and wanted to have this area excised. Surgical Technique: The patient was lying down supine. The left breast was scrubbed with Betadine scrub and paint and draped in the classical fashion. The patient has a transverse incision near where we are feeling this lump, which was over about the 11 o’clock position, high up in the superior aspect of the left breast. A transverse incision was made underneath the breast tissue and adipose tissue was completely taken out. Hemostatic was ascertained with electrocoagulation. The wound was closed using interrupted 3-0 Vicryl sutures, the skin was closed with subcuticular running 5-0 Dexon. Benzoin and Steri-Strips and a pressure dressing were applied. All counts were normal. It was the impression of the pathologist that it represented a benign process in the left breast. 19000–LT19020–LT19120–LT19125–LT28.Emergency Department NoteThe patient was seen in the ED with a staple embedded in the left index finger. An automatic staple gun impaled a staple into the DIP joint. A 1% lidocaine digital block was performed, and incision was made into the joint and the staple was removed with no complications. He was told to watch for any red streaks, swelling, pain or pus.20103–LT26070–LT26075–F626080–F6Feedback: 54150–52 Note that MD used device (hemostat) to crush the foreskin. CPT Guidelines state that modifier 52 is reported when the circumcision (code 54150) is performed without dorsal penile or ring block. 29. Operative Report Preoperative Diagnosis: Abnormal uterine bleeding Postoperative Diagnosis: Same Procedure: Diagnostic hysteroscopy with D&C There was an approximately 8-mm polyp of the cervix. The remainder of the endocervix was unremarkable. Uterine cavity was somewhat difficult to visualize but no obvious abnormalities. Minimal tissue on D&C. Patient was taken to the OR with an IV in place, received general anesthesia and was placed on the operating table in semi-dorsolithotomy position with her legs held by staff. She was then prepped and draped. Pelvic exam was performed. Weighted speculum was placed and single tooth tenaculum placed anteriorly on the cervix. Visualization was good. Diagnostic hysteroscopy was introduced into the endocervix on direct visualization and into the intrauterine cavity. The above findings were noted with no obvious pathology. This was withdrawn and cervix dilated to #8 Hagar. Sharp uterine curette was introduced and the uterine cavity systematically curetted with minimal amount of tissue. Bleeding was negligible and procedure was terminated. Patient tolerated the procedure well and was taken to the recovery room in good condition. Estimated blood loss 15 cc. 5855858558, 581205750058563Feedback: 58558 Note that the D&C is included; do not assign an additional code, it would be “unbundling.” 30.Operative Report Preoperative Diagnosis: Laceration of nerve and tendon, left 5th digit Postoperative Diagnosis: Ulnar nerve laceration, no tendon laceration, left 5th digit Operation: Repair of ulnar nerve Procedure: The patient was brought into the operating room and prepared and draped in the usual sterile manner. A tourniquet was used and inflated to approximately 250 mm of mercury after exsanguination of the hand. Tendons were noted to be completely intact. The nerve was then isolated in both proximal and distal ends and with the use of an operating microscope 9-0 sutures were placed in the epineurium, six through and through sutures placed. When this was finished the nerve was checked for congruity. It should be stated that the nerve was trimmed and the fascicles were lined up end to end as best as possible. After this, copious irrigation was undertaken and bleeders were cauterized. The skin was then closed with 5-0 nylon and a sterile dressing was applied. The tourniquet was let down and a clam digger splint with a rubber band through the nail was placed to ensure range of motion. The patient was discharged to recovery room without complications and there was approximately 15 cc blood loss. No blood replacement. 400 cc of Ringers lactate was used in the case. a.64836–F4b.64836–F4, 69990c.64831–F4d.64831–LT, 6999031.Clinic Record Procedure: Laryngoscopy This 45-year-old patient is seen in the ENT clinic for a chronic sore throat. The patient's mouth is open wide and the tongue held down with a tongue depressor. The laryngeal mirror is inserted into the back of the mouth just above the uvula. I was able to visualize the epiglottis, larynx, and vocal cords. On the larynx appeared a small lesion. The patient is advised to have the lesion removed in the Outpatient Surgery Department on Tuesday. 31505 31512315253157532.Emergency Department Record This 3-year-old male was carried into the ED by mother who states, “a fish tank fell over on him” cutting his forehead and cheek. No LOC, PERRTL: patient alert and oriented. Patient has a 3 1/2 cm superficial laceration over the right eye across forehead and 1-1/2 cm superficial laceration on right cheek. Local anesthesia administered, wound irrigated and sutured with 6-0 nylon. 12011, 12013120521201312051, 1205233.Operative Note: Excision of epidermal benign lesion on dorsum hand measuring 1.9 cm x 0.5 cm x 0.8 cm.The hand was prepped and draped in the usual fashion after obtaining satisfactory analgesia with infiltration of local anesthesia. An elliptical skin incision was made surrounding the lesion. The skin lesion was completely excised and closed with interrupted 4-0 Dexon for the subcutaneous tissue and skin with 4-0 Dexon. Neosporin ointment was applied.1130811402114221142434.Operative Report Preoperative Diagnosis: History of Colon Polyps Postoperative Diagnosis: Polyp of Colon Procedure: Colonoscopy and polypectomy Indications: The patient is a 46-year-old who had a polyp removed a little over a year ago and presents for a follow up at this time. Findings: The patient was taken to the Procedure Room and placed in the supine position. The patient was given initially 50mg of Demerol and 3mg of Versed. Next, a rectal exam was performed and the scope was introduced. The prep was poor. The scope could be passed up to an area of about 35 cm and a polyp was found. It was removed with a snare and then brought out with the biopsy forceps through that port. This specimen was sent to the Pathologist for further evaluation. The scope was brought around to the ascending colon. I could not get the scope to any further. I could not find any gross pathologic changes. The patient received an additional Demerol and Versed during the procedure to a total of 75 of Demerol and 9 of Versed. The scope was then carefully withdrawn and the puddles of fluid were evacuated as the scope was withdrawn. Good hemostasis was found at the site of the polypectomy. The scope was then carefully withdrawn. The patient tolerated the procedure reasonably well. There were no complications. The patient left the Procedure Room in stable condition. Follow up: The patient will follow up in my office in 7 to 14 days. The patient will be given a prescription for Anusol suppositories. 453384538545385–5245388 35.Operative Report Preoperative Diagnosis: Right initial inguinal hernia and umbilical hernia Postoperative Diagnosis: Same Procedure: This 78-year-old patient was taken to Surgery, where he was prepped and draped in the normal sterile fashion. Incision was made from 2 cm above the pubic tubercle toward the anterior iliac spine and deepened to the external oblique. The external oblique was opened. The patient’s cord was elevated on a Penrose drain. He had a very large direct inguinal hernia, no indirect hernia. All of the areas were freed up, and a piece of mesh was designed in a keyhole fashion and sutured in place with 2-0 Prolene, avoiding the nerve. Irrigation was performed. The external oblique was closed with 2-0 running chromic. Irrigation was performed again. Scarpa’s fascia was reapproximated using 3-0 chromic, and the skin was closed with staples. The umbilical hernia was then dissected out after an incision was made beneath the umbilicus. The hernia sac was removed. The fascia was closed with figure-of-eight sutures of 0 Prolene. 2-0 chromic was used to tack down the skin and also reapproximate the subcutaneous area. A running subcuticular of 4-0 Vicryl was placed and Benzoin and Steri-Strips were applied. A dry sterile dressing was applied to each. Betadine was applied to the hernia. The patient was returned to the recovery room in stable condition. 49505–RT, 4958549520–RT, 4958549505–RT, 49585, 4956849507–RT, 49585, 49568Feedback: implantation of mesh CPT code (49568) is only used with incisional or ventral repairs. 36.Operative Report Preoperative Diagnosis: Osteomyelitis, fifth metatarsal, left Postoperative Diagnosis: Same Procedure: Amputation of toe The patient was brought to the operating room and placed in supine position. After adequate general anesthesia was obtained, the left foot was scrubbed, prepped, and draped in the usual manner. No tourniquet was utilized. A skin incision was made along the lateral border of the fifth metatarsal and carried down to the subcutaneous tissue in line with the skin incision. Bleeders were clamped and electrocoagulated. Dissection was carried down to the base of the fifth metatarsal where an osteotomy was made at the base. The bone was then delivered from the wound and sent to the pathology department. There was erosion of the head of the fifth metatarsal consistent with osteomyelitis. The toe was amputated and the entire specimen was sent to the pathology department. All of the tissues were débrided. The wound was irrigated and hemostasis assured. The subcutaneous tissue was very loosely reapproximated utilizing 4-0 Vicryl suture. The skin was not closed and was allowed to drain. A sterile dressing was applied to the wound. The patient was then transferred to the recovery room in satisfactory condition. a.28805–T428810–LT 28810–T428820–LT37.Operative NotePreoperative Diagnosis: Painful left wristPostoperative Diagnosis: Closed distal radial fractureOperation: Closed reduction of left wristUnder satisfactory general anesthesia the patient was placed in supine position. The hand was secured with traction. The fracture was reduced and the alignment was checked with imaging. 25505–LT25600–LT25605–LT25622–LT38.Operative Report Preoperative Diagnosis: Chronic laryngitis with polypoid disease Postoperative Diagnosis: Same Procedure: Laryngoscopy with removal of polyps After adequate premedication, the 60-year-old female patient was taken to the operating room and placed in supine position. The patient was given a general oral endotracheal anesthetic with a small endotracheal tube. The Jako laryngoscope was then inserted. There were noted to be large polyps on both vocal cords, essentially obstructing the glottic airway when the tube was in place. The polyps appeared larger on the right cord. Using the straight-cup forceps, the polyps were removed from the left cord first. The polyps were removed from the right cord up to the anterior commissure. There was very minimal bleeding noted. This opened up the airway extremely well. The patient was extubated and sent to recovery in good condition. 3151231530315403157839. Operative Report Preoperative Diagnosis: Right hydrocele Postoperative Diagnosis: Right spermatocele Operation: Right spermatocelectomy Indications for Procedure: This 54-year-old male has a history of right-sided scrotal enlargement. Scrotal ultrasound preoperatively was consistent with right hydrocele. Operation: The patient was brought to the operative suite, placed in supine position and general anesthesia was administered. His scrotum was shaved. He was then sterilely prepped and draped in the usual manner. A transverse incision across the right hemiscrotum was then made approximately 3.5 cm in length using electrocautery to further dissect this area. The right-sided fluid sac was then exuded from the right hemiscrotum. It seems to be a right spermatocele. Using meticulous care and caution, the spermatocele was divided from the testicle and the vas deferens was identified. There as a moderate degree of difficulty as the spermatocele had separated the epididymis from the patient’s right testicle. So using meticulous care, this was divided free from his spermatocele. The spermatocele was handed off intact to the scrubbed personnel. Hemostasis was achieved. The epididymis was then re-attached to the testicle. The testicle was then replaced into the right hemiscrotum. The wound was closed using a #2-0 locking running chromic stitch and the superficial skin was closed in a horizontal mattress fashion. Patient tolerated the procedure well and was sent to recovery in satisfactory condition. Pathology Report: spermatocele a.55040b.55040, 54840c.54840d.5483040.Operative Report Preoperative Diagnosis: History of recurrent foreskin infection Postoperative Diagnosis: Same Procedure: Circumcision Indications: The patient has had some evidence of recurrent foreskin infection and his wife has had recurrent infections and her gynecologist recommended that Mr. K. undergo circumcision. The patient presented at this time to complete that recommendation. Procedure: The patient was taken to the Operating Room and placed in supine position. General anesthetic was initiated. After good anesthesia was achieved the patient’s penis was prepped and draped in the appropriate fashion. A straight hemostat was used to crush the foreskin on the dorsal aspect first. After it had been placed for a period of time the hemostat was released and the crushed segment was then divided. A similar action was performed on the ventral side. This was done down to the desired site of the circumcision. Then a #3-0 chromic suture was placed on the dorsum ventral side connecting the cut ends of tissue. Curved hemostats were used circumferentially around the penis on the right side to the desired length of circumcision. After the tissue was crushed it was divided and then the excess foreskin was removed. Good hemostasis was achieved using the Bovie and the remaining cut ends of the tissue were reapproximated using interrupted #3-0 chromic suture. Similar action was done on the left side. Remaining cut edges of the tissue were reapproximated using interrupted #3-0 chromic sutures. Vaseline gauze was placed at the suture line followed by dry gauze. The patient tolerated the procedure well. There were no complications. The patient left the Operating Room in stable condition. Follow up: The patient will follow up in my office in 7 to 10 days. He was given a prescription for Darvocet N 100 mg. 5415054150–525416054161Chapters 5, 6, 8, 9 Radiology, Pathology and Laboratory, Medicine, Anesthesia This lesson will focus on key concepts in the following topics: Radiology Pathology and laboratory Medicine Anesthesia Objectives 1.Radiology a.Apply modifiers associated with radiological procedures (26, TC) b.State the meaning of the phrase “supervision and interpretation” as it applies to radiological procedures 2.Laboratory a.Apply physician-billing guidelines for coding laboratory services b.State the appropriate use of organ- or disease-oriented panels c.Select pathology level codes associated with pathological examination and diagnosis 3.Medicine a.Assign codes to report the administration of vaccines or toxoids b.Apply guidelines to identify injections and infusions 4.Anesthesia a.Apply “qualifying circumstances” and “physical status” codes to anesthesiology services b.Given a procedure/service, successfully assign CPT code(s) for the above listed services (radiology, pathology and laboratory, medicine, and anesthesia) Suggested Student Activities and Assignments 1.Research coding references available from specialty professional organizations. 2.Assign the students a specialty area to present to the entire class. 3.Obtain a printout of chargemaster files or superbills and assign the students a quality review exercise. Key Points for Lecture Notes 1.This lesson covers radiology, pathology and laboratory, medicine, and anesthesia (chapters 5, 6, 8, and 9 in Basic Current Procedural Terminology and HCPCS Coding). 2.Because many of the services covered in this lesson are billed through the use of a chargemaster or super bill, coders do not frequently assign the specific codes. However, the coder should be aware of the pertinent conventions as they apply to each section of the CPT book. 3.After graduation, if a student is employed by a radiologist, anesthesiologist, or pathologist then they will need a more in-depth lesson for these sections. Also, note that many professional organizations (for example, the American Society of Anesthesiologists) provide supplemental materials for CPT coding with their specific specialty. Chapter 5, 6, 8, and 9 Test Bank Items Chapter 5 Radiology Test Bank Items1.True or FalseThe patient undergoes an ultrasound of the gallbladder. The correct code assignment is 76700. FalseFeedback: The correct code is 76705.2. The radiological exam included two views of the mandible. The correct code assignment is 70100.True3.A patient undergoes a retrograde urethrocystogram. The same physician performs both the injection and the supervision and interpretation. What is the correct CPT code assignment for this physician?a.51610, 74450–26 b.51610, 74450 c.74450–26 d.74450 4.A single-view, frontal X-ray of the chest was taken and the radiologist provided only the supervision and interpretation for the procedure. What is the correct CPT code assignment for the radiologist’s services? a.71010 b.71010–TC c.71010–26 d.71015 5.A patient is seen in the Emergency Department after falling and injuring his elbow. A CT scan is performed for evaluation.732007320173206732186.True or FalseA MRI of brain (without contrast material) was performed to rule out the diagnosis of cerebral vascular accident. The correct code assignment is 70551.True7.The patient undergoes MRI of the pelvis, first with no contrast, and then followed by contrast material.72192, 729137219472195, 7219672197True or FalseThe radiologist performed an angiography of the patient’s arm, with the focus on evaluation of the AV fistula. The correct code assignment is 75710.FalseFeedback: The correct code is 75791.The radiologist performed an MRI, without contrast, of the patient’s knee. The correct code assignment is 73721.TrueThe radiologist provides only the supervision and interpretation of a hysterosalpingography. What is the correct CPT code assignment for the radiologist? a.74740 b.74740–26 c.58340, 74740 d.58340, 74740–26 Chapter 6 Pathology and Laboratory Test Bank Items1.The pathologist performed a gross and microscopic examination of a kidney biopsy. What is the correct CPT code assignment? a.88300, 88305 b.88305 c.88307 d.88300, 88307 2.A physician draws blood to test for levels of T3 on a non-Medicare patient. The blood is sent to an outside laboratory for analysis. When billing for the physician’s services, which of the following modifiers should be appended to CPT code 84480? a.52 b.59 c.90 d.91 3.A physician orders part of a Hepatic Function Panel: Serum Albumin, Total Bilirubin, Direct Bilirubin and SGPT, SGOT. What is the correct CPT code assignment? a.80076 b.80076–52 c.82040, 82247, 82248, 84155, 84460, 84450 d.82040, 82247, 82248, 84460, 84450 4.True or FalseThe physician orders the patient to have a lipid panel to include: total serum cholesterol, triglycerides, and HDL levels. The correct code assignment is 80061.True 5.True or FalseThe correct code assignment for a Gross and microscopic examination of a wedge biopsy of the lung is 88305.FalseFeedback: The correct code is 88307. Chapter 8 Medicine Test Bank Items1.What is the correct code assignment for bilateral EMG of cranial nerves? a.95867 b.95867–50 c.95868 d.95868–50 2.What is the correct CPT code assignment from the Medicine chapter for IM injection of Leukine?a.96401b.96372c.96374d.96369Feedback: An additional J code would be assigned to identify the specific drug. Students will have to determine the purpose of Leukine (prevent infection in cancer patients) before assigning the correct code. 3. True or FalseA 62-year-old patient is seen in the outpatient hemodialysis clinic for 3 face-to-face visits in the month of July for treatment of end-stage renal disease. CPT code 90961 is assigned.True True or FalseThe patient receives an evaluation of auditory rehabilitation status, 1 1/2 hour visit. CPT codes 92626 and 92627 are assigned.False Feedback: Codes 92626, 92627, 92627 should be assigned.5.True or FalseThe patient receives a 30-minute IV infusion of 2 g of Rocephin. In addition to the J code, the CPT code 96365 should be assigned. True6.True or False The 34-year-old patient receives an administration of tetanus toxoid by IM injection. The correct code is 90460, 90703FalseFeedback: The correct code assignment is 90471, 90703. 7.True or FalseThe physician performs chiropractic manipulation treatment of three spinal regions. The correct code assignment is 98941. True Chapter 9 Anesthesiology Test Bank Items1. True or FalseThe 35-year-old patient undergoes an incisional hernia repair (lower abdomen) and the anesthesia code is 00830.FalseFeedback: The correct code is 00832.2.The patient received anesthesia for an open reduction of a fracture of the head of the femur. What would be the correct CPT code assignment for the anesthesiologist’s services?013400136001390013923.True or FalseAnesthesia is provided for a patient that is having a reverse shoulder replacement. The correct code assignment is 01634. FalseFeedback: The code should be 01638 4.A patient undergoes a modified radical mastectomy. What would be the correct CPT code assignment for the anesthesiologist’s services? a.00474 b.00406 c.00404 d.00400, 00406 5.The patient received anesthesia for an open reduction of a fracture of the head of the femur. What would be the correct CPT code assignment for the anesthesiologist’s services?013400136001390013926. True or False. The patient receives anesthesia for repair of cleft palate. CPT code 00172 is reported. TrueChapter 7 Evaluation and Management Services This chapter will focus on the following topics: Documentation guidelines New versus established patient Components of E/M services Objectives After completing this chapter, the student should be able to do the following: 1.Describe the contents of the evaluation and management (E/M) services section 2.Differentiate between a new and established patient 3.Identify the seven components included in the descriptors for the levels of E/M services 4.List the three key components for levels of E/M services 5.Describe the documentation requirements for E/M key components 6.State the circumstances when time is a key factor in determining E/M level of service 7.Given a scenario, assign the appropriate E/M code Suggested Student Activities and Assignments 1.Review the AMA website for current information about documentation guidelines. 2.Research topic on Internet including new proposal for facility E/M guidelines. 3.Invite guest speakers from physician’s office settings. Key Points for Lecture Notes 1.Introduction a.This unit is very complex and difficult to understand at first glance. For this reason, the chapter is at the end of the textbook. b.evaluation and management (E/M) services are the most common codes used in the physician’s office. In the hospital setting, E/M codes are used to describe the services for the emergency department visits for the facilities’ cost in providing care. Traditionally, hospital coders have little experience with E/M coding because these services are part of the computerized chargemaster file and the emergency department (ED) physician or staff takes the responsibility for deciding the correct code assignment. This has changed with the implementation of the APC system. 2.E/M in the beginning the CPT book. a.E/M codes were introduced in 1992, but came with no documentation guidelines. Clinical Examples (appendix C) gave the physicians some guidance, but it was not until 1995 that AMA and HCFA provided official documentation guidelines. Revised guidelines were published in 1997. Currently, physicians can use either the 1995 or 1997 guidelines. 3.E/M services. Ask the students to read the guidelines and definitions for E/M coding and to become familiar with the format and organization. Selection of the appropriate code is based on: Type of service, such as: office visit, consultation, preventive medicine Where service provided, such as: office, hospital, skilled nursing facility Extent: physician document should support the work involved with the case 4.New versus established patient. A glance at the descriptions in the beginning of the E/M section will reveal the categories of new and established patients. The criterion for determining whether a patient is new is based on a 3-year rule. If the patient has not received professional services (face-to-face) by his physician or any other physician of the same specialty (or subspecialty) in the group, then the patient is considered new. If the patient has received services, then the patient is established. Consider the following two scenarios and ask the students to determine if the patient is new or established: Case #1: Office Visit (1/11/14) Dr. Smith --Patient last seen by this primary care physician (Dr. Smith) on 12/22/12 for strep throat. Patient is now seen for dermatitis of unknown origin. Dr. Smith elects to refer this patient to another physician in his group practice who is a dermatologist (Dr. Jones). Case #2: Office Visit (1/12/14) Dr. Jones Patient treated for dermatitis. Answer: On the first visit at Dr. Smith’s office, the patient would be considered established since the patient was treated less than 3 years ago. Dr. Jones’ visit would be considered new since a different specialist in the group practice saw the patient. If Dr. Jones had been another primary care physician, then the patient would have been considered established even though Dr. Jones had never seen the patient before. Refer students to the decision tree in CPT for new vs. established patient.5.Key components. This is where the process gets complicated. The three key components are: History Examination Medical decision making Point out that the above-mentioned components drive the selection process. Review of the E/M codes reveals such terminology as: problem-focused, expanded problem-focused, detailed, and comprehensive. E/M codes start with an uncomplicated type of visit (for example, dressing change) to the most comprehensive case (chest pain). The physician’s documentation must support the level chosen. AMA/CMS documentation guidelines state exactly how many of the elements must be documented to qualify for the different levels. It is an attempt to quantify something that is not easily quantified. Another underlying key consideration is medical necessity. If the patient comes to the physician because of an ingrown toenail, a high level of service would not be appropriate. The same would be true of a patient seen with severe abdominal pain. You would expect a high level of E/M code (99214 or 99215) not a low level for treatment of severe abdominal pain. 6.Focus on documentation requirements. The PowerPoint presentation and the Chapter Tables outline the elements necessary to support the level of service. 7.Time. Note in the E/M code descriptions the mention of “time.” Time only becomes a factor when counseling or coordination of care dominates (more than 50 percent) the visit. When time dominates the visit, then time becomes the controlling factor for code selection. Example: An established patient is seen by her family physician to discuss management of medications. Counseling and coordination dominates the visit. The physician documents that the visit took 15 minutes, and 10 minutes of the visit was for counseling. Time becomes the key factor, and code 99213 would be assigned. In other words, documentation of history, examination, and medical decision making does not determine the code selection for this case. 8.E/M and APCs. With implementation of APC methodology, hospitals were required to develop a system for consistent assignment of E/M codes to represent the facilities’ services. Most hospitals have created a point system to systematically access the resources necessary to treat the patient. 9.A consultation initiated by a patient or family (not by physician) is NOT assigned a code from the Consultation section. In this case, coders are referred to other CPT sections.Chapter 7 Test Bank Items 1.Patient is seen by her primary care physician for headaches. The physician performs a physical exam, reviews data, and outlines management options. Which of the following key components is missing from this case? a.Time b.History c.Medical decision making d.Coordination of care 2.Skilled Nursing Home Visit Date of service: 1/9/14 Last date of treatment: 12/22/13 Physician visits and elderly patient in the skilled care facility. Physician performs a detailed interval history, comprehensive examination, and medical decision making is of moderate complexity. In addition, the physician reviewed the medical record and the recent lab results. What is the correct E/M code for this service? 99305993069930999310 3.Which of the following is a key factor for selecting a correct code for preventive medicine services? a.Physician time b.Coordination of care c.Age of patient d.Review of systems 4.Office Visit Date of service: 1/3/14 Last date of treatment: 2/12/11 The patient is seen for a cough and sore throat. The physician performs a problem-focused history, expanded problem-focused examination, and medical decision making is straightforward. What is the correct E/M code for this service? 99201992029921199212 5.True or FalsePhysician Office Record Physician monitors the management of a patient who it taking long-term warfarin therapy. During this initial 90-day period, the physician monitors the dosage with appropriate testing. The correct code assignment is 99363. True6.A patient is seen in the emergency department with a severe headache that is not responding to over the counter medications. The physician performs a detailed history and examination with medical decision making of high complexity. What is the correct E/M code for this encounter?a.99282b.99283c.99284d.99285Feedback: the level of service requires all 3 of the key components.7.Office Visit Date of service: 9/28/14 Last date of treatment: 8/3/10 The patient is seen for a chief complaint of shortness of breath and fatigue. The physician performs a detailed history, comprehensive examination, and medical decision making is of moderate complexity. What is the correct E/M code for this service? 9920399204 9921399214Feedback: For a New patient the History would have to be comprehensive for assignment of 99204 8.The physician conducts a home visit for an established patient who is bed-ridden. A comprehensive interval history and comprehensive examination is performed with medical decision making of moderate complexity. What is the correct E/M code for this encounter?993449934599349993509. Rolling River Community Village-Patient Visit Physician sees a new patient in the independent living area of this retirement community. The physician performs a detailed history, expanded problem-focused examination and medical decision making is of moderate complexity. What is the correct E/M code for this service? 9931899324993259932610.True or FalseA patient was seen by his family practitioner two years ago. A cardiologist in the same group practice now sees this patient for the first time. For E/M code selection for the cardiologist, the patient would be classified as new.True11.True or FalseOffice Visit Date of service 11/24/14 Last date of treatment: 7/12/13 The patient is seen for a routine blood pressure check. Nurse documents BP: 135/90. Nurse asks about diet and exercise program. Patient offers no complaints. The correct E/M code for this service is 99211.TrueChapter 10 HCPCS Level II This chapter will focus on the following topics: Guidelines for use of HCPCS Level II HCPCS Level II modifiers Objectives 1.Identify the structure of HCPCS Level II codes 2.Successfully apply general guidelines for HCPCS Level II coding assignment 3.Describe the use of HCPCS Level II modifiers 4.Identify the HCPCS Level II modifiers required for hospital outpatient services Suggested Student Activities and Assignments 1.Review Medicare Bulletins for HCPCS Level II coding guidelines. 2.Review superbills for use of Level II codes. 3.Faculty may elect not to have students purchase the HCPCS Level II code book but instead use an encoder or place a copy on reserve in the library. In addition, the following link provides an electronic version of HCPCS codes: Points for Lecture Notes 1.Introduction. Take time to review the contents of the HCPCS Level II code book. The Centers for Medicare and Medicaid Services (CMS) revises Level II codes every year. Note that the exercises in this guide are from the 2012 edition of HCPCS. anization. Level II codes are alphanumeric (letters A–V). Review with the students the following characteristics: a.D codes are copyrighted by the American Dental Association. b.C codes are temporary codes for use with outpatient prospective payment only. c.G codes are also temporary codes that CMS assigns for procedures and services that are being considered for inclusion in CPT. Example: G0105 Colorectal cancer screening: colonoscopy on individual at high risk d.Similar to CPT, there is an alphabetic index for locating codes. e.Level II codes are part of the HCPCS coding system. f.Because CPT lacks many codes for non-physician procedures and services, CMS created codes to supplement CPT. 3.National modifiers. In addition to the five-digit alphanumeric codes, HCPCS Level II also contains modifiers. The modifiers are two digits and are either alphanumeric or strictly alphabetic. Chapter 10 Test Bank Items Note that the following answers are from the HCPCS Level II 2003 edition. Unless otherwise noted, assume all services and supplies are for Medicare patients. 1.What is the correct HCPCS code for a patient who requires a silicone gel, removable foot insert for both shoes?L3001L3001 x 2L3003L3003 x 22.True or FalseThe two-year-old child required a surgical boot. HCPCS code L3208 would be reported for this equipment.FalseFeedback: The correct code is L3209.3.The patient requires a heavy-duty wheeled walker, what is the correct HCPCS code for this equipment?E0135E0148E0149E01584.What is the correct HCPCS code for 50 reagent strips for home blood glucose monitoring device?A4250A4252A4253A42555.The patient requires a new battery for his hearing aid. What HCPCS code represents this equipment?V5242V5264V5266V52676.A patient requires an injection of 40 mg of methylprednisolone. What is the correct HCPCS code?J1020 J1020 x 2J1030J10407.What is the correct HCPCS code for below the knee surgical stockings for both legs?a.A4490 x 2b.A4500c.A4500 x 2d.A4510 8.The nutritional specialist provides 60 minutes of outpatient training for a patient who requires management of their diabetes. What is the correct HCPCS code to represent this service?G0108G0108 x 2G0109G0109 x 29.True or FalseThe patient requires a sleep apnea monitoring with a recorder. This equipment would be reported with HCPCS code E0618.FalseFeedback: The correct code is E0619.10.What is the correct HCPCS code to report a patient receiving an injection of 250 mg of Aminophyllin?a.J0280b.J0285c.J0290d.J0295Chapter 11 Reimbursement in the Ambulatory Setting This chapter will focus on the following topics: Reimbursement in ambulatory settings Medicare outpatient code editor Quality review Objectives 1.Explain the prospective payment system utilized for Medicare reimbursement for ambulatory surgery procedures 2.Identify the characteristics of the resource-based relative value scale (RBRVS) for Medicare reimbursement for physicians 3.Identify the characteristics of ambulatory patient classification (APC) groups 4.Describe the purpose of the Medicare outpatient code editor 5.Define unbundling 6.Apply coding skills to detect errors in submitted data Suggested Student Activities and Assignments 1.Research current information about prospective payment system. 2.Go to the Centers for Medicare and Medicaid Services (CMS) website: . Determine the names and addresses of the Medicare Part A fiscal intermediary and Part B carriers for students’ state. Access the websites for both carriers. 3.Invite guest speakers to discuss the impact of the outpatient prospective payment system. 4.Develop encoding project (using Exercise 11.1) so that students can see the impact of their coding decisions. 5.Discuss articles from the AHIMA Body of Knowledge, such as: “Care and Maintenance of Chargemasters,” AHIMA Practice Brief, March 30, 2010.Exercise 11.1 provides students the opportunity to code diagnoses using ICD-10-CM. Key Points for Lecture Notes 1.Introduction. This unit provides an overview of the use of CPT codes in the reimbursement process. In addition, part of the unit is devoted to ongoing quality improvement of coded data. 2.Reimbursement. The purpose of this chapter is to provide an overview of the reimbursement systems that utilize CPT and HCPCS coding systems. The textbook does not provide an in-depth explanation of the ambulatory payment classification (APC) system or the RBRVS for physician services. The text provides definitions of common terms and examples to allow students to recognize the connection between coding and reimbursement. Students should be encouraged to use the CMS website to research current memoranda pertaining to reimbursement and coding. 3.Quality of coded data. Quality of coding is a major concern of health care providers and insurance carriers alike. The exercises provided in the textbook will allow the students to practice their skills of reviewing previously coded data. Ask the students to compare the assigned codes with the guidelines provided in the textbook to determine errors. Remind the students that coded data must be supported by documentation. 4.Unbundling. Unbundling is the practice of coding services separately that should be coded together as a package because all parts are included within one code and, therefore, one price. Unbundling done deliberately to obtain higher reimbursement by using the unbundled codes is a misrepresentation of services and can be considered to be insurance fraud. 5.Typical types of errors a.Failure to review the entire record for supportive documentation b.Selection of incorrect code c.Data entry errors d.Assigning codes that are not validated by documentation 6.Suggestions for improving the quality of coding a.Maintain current coding policy and procedure manual b.Address process for reviewing the health record c.Require use of coding references d.Provide ongoing education for coding staff e.Develop a procedure for clarifying ambiguous documentation f.Perform ongoing auditing 7.Transition to ICD-10-CMAn optional activity for coding ICD-10-CM for diagnoses has been added to Chapter 11 of Basic CPT. The students can use the electronic files located on the CDC website: 11 Test Bank Items True FalseQuality Review of Coded Data. The following are excerpts from HCFA-1500 billing forms containing ICD-10-CM diagnosis codes linked to HCPCS codes. In each problem, verify the CPT code assignment. The questions are formatting to indicate True if it coded correctly and False if it is not the correct code assignment. The ICD-10-CM codes are provided for reference only. 1. An established patient seen in the physician’s office for sore throat and a temperature. The physician performed a problem-focused history, problem-focused examination and medical decision making was of low complexity. The final diagnosis was acute pharyngitis. FalseFeedback: The documentation supports code 99212.The patient was diagnosed with a suspicious left breast mass. Under ultrasound guidance, percutaneous, the surgeon inserted a breast marker and performed a biopsy.FalseFeedback: The correct code is 19083 (ultrasound guidance)3.The patient has been experiencing occasional rectal bleeding. The surgeon performed a colonoscopy that extended to the cecum. A biopsy was taken of tissue in the ascending colon, the source of the bleeding was not found.FalseFeedback: The only code that should be reported is 45380. The surgeon did not perform any procedure related to the rectal bleeding.The physician documents the diagnosis as calculus of the ureter. The surgeon performs a cystoscopy, ureteroscopy for fragmentation of the stone and insertion of double J stentTrue5.The patient was taken to the outpatient surgical suite with the diagnosis of chronic hoarseness. The surgeon performed a flexible bronchoscopy with bronchial cell washings and brushings.True 6.Patient with menorrhagia had a laparoscopic lysis of adhesions of fallopian tube and excision of benign tumor of ovary.True7.A patient suffers an abdominal aortic aneurysm. Under fluoroscopic guidance, the surgeon inserts a modular bifurcated prosthesis using one docking limb. FalseFeedback: Correct code is 348028.The physician repaired the 2.0 cm superficial laceration of the forehead and 2.5 cm laceration of the scalp with simple wound closures.FalseFeedback: Correct codes are 12011 & 12001. These two wound repairs cannot be added because they are not from the same anatomic site code description. 9.Physician excises a 2.0-cm lesion (basal cell carcinoma) from the patient’s left arm. The excised margins extended 0.5 cm from around the lesion. A simple repair is used to close the wound. FalseFeedback: CPT code 11403 is excision of benign lesion and this case specifies excision for a malignant lesion. Correct code assignment: 11604.10.Patient is being treated for a spontaneous abortion and the physician performs a D&C. FalseFeedback: 58120 is for a nonobstetric D&C, correct code is 59812Full Answer Key to Textbook 2016(including Appendix B)Chapter 1 Answer KeyChapter 1: Introduction to Clinical CodingChapter 1 Review1.The American Medical Association (AMA) updates the CPT codes.2. Centers for Medicare and Medicaid Services (CMS)3. Diagnosis code set (ICD-10-CM)4.a.ICD-10-CMb.CPTc.CPT5. All of the diagnoses except G83.84 Todd’s paralysis would support medical necessity.6. C . 87172 Pinworm exam_ 7.Procedure code 11440 is linked with diagnosis code A (D22.30). Procedure code 82951 is linked with diagnosis code B (R73.9).Note: Depending on the carrier, you may link more than one reference number in block 24E, whereas some payers require just one. When reporting more than one code on a CMS-1500 claim, enter the code with the highest fee in line 1 of block 24 and the remaining in descending order of charges.Chapter 2 Answer KeyChapter 2: Application of the CPT SystemExercise 2.1 Organization of CPT?1.Anesthesia, Perineum (Category I)?2. Radiology, Diagnostic Radiology (Diagnostic Imaging) (Category I)?3.Pathology and Laboratory, Drug Assay (Category I)?4.Category II code?5. Evaluation and Management, Hospital Inpatient Services (Category I)?6. Medicine, Cardiovascular (Category I)?7. Surgery, Digestive (Category I)?8.Category III code?9. Surgery, Maternity Care and Delivery (Category I)10. Evaluation and Management, Hospital Observation Services (Category I)11. Radiology, Nuclear Medicine (Category I))12. Pathology and Laboratory, Surgical Pathology (Category I)Exercise 2.2 CPT?Conventions?1. No ? 2. Two?3. New descriptor?4. Repair initial incisional or ventral hernia; incarcerated or strangulated?5.Revised descriptor?6.Repair blood vessel, direct; hand, finger?7.No?8.31578, 31576 9.4081410.13101, 13102Exercise 2.3 Use of the Alphabetical Index?1.23400Green operation, see scapulopexy?2.49550Hernia repair, femoral, initial ?3.35556Bypass graft, venous, femoral-popliteal ?4.10021 Fine needle aspiration, diagnostic ?5.26991Incision and drainage, bursa, hip?6.01472Anesthesia, achilles tendon repair tendon repair?7.31526Laryngoscopy, direct, diagnostic?8.95955EEG, see Electroencephalography, intraoperative ?9.11055Paring, skin lesion, benign hyperkeratotic 10.20605Bursa, elbow, aspiration 11.41105Biopsy, tongue 12.43251Esophagogastroduodenoscopy, flexible transoral, removal, polyp 13.29881Arthroscopy, surgical, knee 14.72196MRI see magnetic resonance imaging, (MRI), diagnostic, pelvis 15.65222Removal, foreign body, cornea, with slit lamp 16.58356Endometrial Ablation, cryoablation 17.4008FBeta-blocker therapy, see Performance Measures, interventions – Performance measures, coronary artery disease, anti-hypertensive agents, Beta-Blocker therapy 18.0184TTransanal endoscopic microsurgery (TEMS), see Rectum, Tumor, Excision, Transanal Endoscopic19.59001Amniocentesis, therapeutic, amniotic fluid reduction 20.87340 Hepatitis Antigen, detection, immunoassay, B Surface (HBsAg)Exercise 2.4 CPT Coding ProcessOperative Report #1?1.Colonoscopy and polypectomy?2.45384, 45385?3.How was the polyp removed (hot biopsy forceps, snare, and so on)??4.“Was removed with hot biopsy forceps and retrieved”?5.45384Operative Report #2?1.Excision?2.Can be located under Excision, lesion, skin; or Lesion, skin, excision. To narrow the selection, coders must determine if the lesion was benign or malignant.Selections:Benign11400–11471Malignant11600–11646?3.Documentation is needed to code malignant lesion, size of lesion + margins (or size of excision)(2.0 cm + 0.5 cm + 0.5 cm = 3.0 cm excision site) and site (arm). ?4.Pathologic diagnosis indicates that the lesion was malignant (11600–11646).?5.11603Operative Report #3?1.Hernia repair?2.Index entries: Hernia repair, umbilicus (child, 5 years and up); Repair, hernia, umbilical. Codes to review:49580–49587?3.Age of patient; incarcerated or strangulated hernia ?4.Review of documentation (38-year-old patient and no documentation of incarcerated/strangulated hernia). Using the abstracted documentation and process of elimination, the correct code would be 49585.?5.Wound closure would be an integral part of the procedure and would not be assigned a CPT code.Exercise 2.5 Coding References?1.CPT Assistant, January 1996, page 7, instructs the coder to assign 45380 or 45385 depending on the actual technique employed. CPT Assistant, January 2004, states that if a small polyp is removed via cold knife biopsy, the appropriate code is 45380. This is a good example of the need to research the most current coding advice. ?2.45380Exercise 2.6 Coding References?1.When a biopsy of a lesion is obtained and the remaining portion of the same lesion is then excised/fulgurated, only the code for the excision/fulguration should be used. When the biopsy is taken from a different lesion than the one excised, the biopsy code and an additional code for the removal of the separate lesion are reported. It would be appropriate to append modifier 59 to the code reported for the biopsy procedure.?2.Reference: CPT Assistant, October 2004, Skin Biopsy Coding Guidelines.Exercise 2.7 Coding References1.No. Code 36479 would be reported only once. The code descriptor for code 36479 states, “second and subsequent veins treated in a single extremity, each through separate access sites,” indicating that the second, third, fourth, etc. vein(s) are represented in code 36479. No additional reporting occurs after the second vein is treated.2.Reference: CPT Assistant, July 2012, pages 1213 (Frequently Asked Questions).3.CPT Assistant, September 2014, page 13 (Frequently Asked Questions). Zone II is defined as proximal half of middle phalanx to metacarpophalangeal (MCP) joint, inclusive of the MCP.4.CPT Assistant, June 2015, page 10 (Frequently Asked Questions). Hot biopsy forceps uses electrocautery, therefore, code 45384 would be the appropriate code to report colonoscopy with polypectomy in which bipolar cautery is the tool used to perform the procedure.5. CPT Assistant, March 2014, page 14 (Frequently Asked Questions). There is no specific CPT code for draining the container; it would be included in the evaluation and management (E/M code).6. CPT Assistant, July 2015, Page 10 (Frequently Asked Questions). It would be appropriate to report code 31299, Unlisted procedure, accessory sinuses, for an endoscopic medial maxillectomy because no other existing code describes the service listed. When reporting an unlisted code to describe a procedure or service, it is necessary to submit supporting documentation (e.g. procedure report) along with the claim to provide an adequate description of the nature, extent, and need for the procedure, and the time, effort, and equipment necessary to provide the service.Chapter 2 Review?1.Bullet?2.No. Versed is a type of conscious sedation. The ?symbol appears before code 43235; therefore, administration or monitoring of conscious sedation would be inherent in the procedure.?3.Category III?4.11730, 11732 ?5.40842?6.Removal of less than 80 percent of vulvar area, and there was removal of skin and deep subcutaneous tissue. (See the note before CPT code 56405.)?7.No. The instructional note after code 43275 states to code once during the same session.?8.11100, 11101, 11101? 9.43260 Diagnostic ERCP10.15200, 15201, 1520111. 2193212. 42320 Chapter 3 Answer KeyChapter 3: ModifiersChapter 3 Review?1. 62?2. 78?3. 22?4.55?5.53?6.19081–RTBiopsy, breast, with imaging of specimen?7.11730–FAAvulsion, nails?8.11043–73Debridement, muscle?9.64611-52Chemodenervation, salivary glands (see note under code 64611) 10.47562-22 Cholecystectomy, laparoscopic11.64721–50Carpal tunnel syndrome, decompression12.28485–RTFracture, metatarsal, open treatmentNote: –T9 is not applicable in this case because the procedure refers to the bones of the foot, not the toes.13.67800-E2Chalazion, singleChapter 4 Answer KeyChapter 4: SurgeryExercise 4.1 Integumentary System—Debridement1.11042 Debridement, skin, subcutaneous tissue2.11005 Debridement, skin, infected11008 Removal, mesh, abdominal wall 3.11011 Debridement, skin, with open fracture and/or dislocation 4.11043 Debridement, muscleExercise 4.2 Integumentary System—Lesions?1.11421Excision, skin, lesion, benign11402Lesion, skin, excision, benign ?2.11643Lesion, skin, excision, malignant?3.11642Excision, skin, lesion, malignant; or Lesion, skin, excision, malignant?4.11200Skin, tags, removal 11201 Each additional 10 lesions, or part thereof?5.11403Lesion, skin, excision, benign?6.17110Lesion, skin, destruction, benign?7.11641Lesion, skin, excision, malignant11640 8. 11056 Lesion, skin, paringExercise 4.3 Integumentary System—Operative Reports?Operative Report #111442Excision, skin, lesion, benign; or Lesion, skin, excision, benign Operative Report #2?11308Lesion, skin, shavingExercise 4.4 Integumentary System – Wound Repairs?1.12034Wound, repair, arms, intermediate ?2.12001Wound, repair; legs, simple ?3.12041Wound, repair, hands, intermediate12002Wound, repair, arms and legs, simple (sum of repairs)Exercise 4.5 Integumentary System ED/Operative ReportsED Report #112001Wound, repair; or Repair, wound, simpleED Report #212032Wound, repair, arms, intermediate, 12005Wound, repair, arms, simpleED Report #313121Wound, repair, scalp, complex 13122Operative Report #411603Lesion, skin, excision, malignant12031Wound, repair, arms, intermediateExercise 4.6 Integumentary System – Skin Grafts?1.15150Skin Graft and Flap, tissue-cultured ?2.14021Skin Graft and Flap, tissue transfer ?3.15220Skin Graft and Flap, free skin graft, full thicknessExercise 4.7 Integumentary System-Operative ReportsOperative Report #115120 Split Grafts11646Excision, skin, lesion, malignant; or Lesion, skin, excision, malignantOperative Report #215100 Skin, grafts, freeExercise 4.8 Integumentary System—Operative ReportsOperative Report #1?19000–LTBreast, cyst, puncture aspirationOperative Report #219125–LTExcision, breast, cyst; or Breast, excision, lesion by needle localizationNote: The hospital also would assign a code from the code range 19281–19287 for placement of the wire. The surgeon would not assign this code because the radiologist performed the procedure.Operative Report #319120–RTBreast, excision, cyst; or Excision, breast, cystNote: The entire nodule was excised, not just a piece of tissue, which is implied with the term biopsy.Exercise 4.9 Integumentary System Review?1.16020Burns, dressings?2.11042Debridement, skin, subcutaneous tissue?3.12032Wound, repair, arms, intermediateNote: Anatomic modifiers (LT, RT) are not appropriate for skin repair.?4.15781Dermabrasion ?5.11606Lesion, skin, excision, malignant?6.11770Pilonidal cyst, excision (Single-layer closure indicates “simple.”)?7.11750–T5Nails, removal?8.19083–LTBreast, biopsy, with localization device placement, ultrasound guidance?9.17273Skin, destruction, malignant lesion10.14001 Skin Graft and Flap, tissue transfer Note: Excision of lesion is included and should not be assigned an additional code.Exercise 4.10 Musculoskeletal System—Fractures?1.25545–LTFracture, ulna, shaft, open treatment?2.24515–LT Fracture, humerus, shaft, open treatment?3.23605–RTFracture, humerus, closed treatment with manipulation?4.27781–LTFracture, fibula, closed treatment (initial cast application included with treatment code)?5.27562–RTDislocation, patella, closed treatment?6.21812Fracture, rib, open?7.24516–RTFracture, humerus, shaftExercise 4.11 Musculoskeletal System— ED/Operative ReportsED Report #1?29515–RTSplint, leg, shortNote: E/M code 99283–25 is applicable in this case.Operative Report #227788–LTFracture, fibula, closed treatment; or Fracture, fibula, shaft, with manipulationExercise 4.12 Musculoskeletal System—Arthroscopy?1.29827–LTArthroscopy, surgical, shoulder?2.29876–LTArthroscopy, surgical, knee 3.29835–RTArthroscopy, surgical, elbow ?4.29846–LTArthroscopy, surgical, wrist Exercise 4.13 Musculoskeletal System—Operative ReportOperative Report #129882–RTArthroscopy, surgical, kneeExercise 4.14 Musculoskeletal System Review?1.28475–LTFracture, metatarsal, closed treatment28475–LT?2.28285–T6Hammertoe repair, correction?3.27769–RTFracture, ankle, medial?4.27372–RTRemoval, foreign body, knee joint ?5.23030Hematoma, shoulder, incision and drainage?6.29844–RTArthroscopy, surgical, wrist ?7.28750–LTArthrodesis, metatarsophalangeal joint, great toe?8.24345–RTLigament, elbow, collateral ligament, repair?9.28108–T2Excision, cyst, phalanges, foot10.20101Wound, exploration, penetrating, chest Exercise 4.15 Respiratory System—Endoscopy?1.31255–50Ethmoidectomy, endoscopic?2.31238–LTEndoscopy, nose, surgical; or Nose, endoscopy, surgical?3.31233–50Sinusoscopy, sinus, maxillary Exercise 4.16 Respiratory SystemOperative Report #130520Septoplasty31267–50Endoscopy, nose, surgical30140–50Turbinate, excisionExercise 4.17 Respiratory System—Laryngoscopy?1.31540Laryngoscopy, direct?2.31577Laryngoscopy, fiberoptic?3.31541Laryngoscopy, direct ?4.31510Laryngoscopy, indirect Exercise 4.18 Respiratory System—Operative ReportOperative Report #131536Laryngoscopy, directExercise 4.19 Respiratory System—Bronchoscopy?1.31640Bronchoscopy, removal, tumor ?2.31628Bronchoscopy, biopsy ?3.31625Bronchoscopy, biopsy31623Bronchoscopy, brushing/protective brushing 4. 31652 Bronchoscopy, with Endobronchial Ultrasound (EBUS)Exercise 4.20 Respiratory System—Operative ReportOperative Report #131623Bronchoscopy, brushing/protective brushingOperative Report #231628Bronchoscopy, biopsy31623Bronchoscopy, brushingsExercise 4.21 Respiratory System Review?1.32663Thoracoscopy, surgical with lobectomy?2.32557Puncture, pleural cavity, drainage ?3.30130Excision, turbinate, inferior ?4.31623Bronchoscopy, brushings (code for cell washings [31622] is a separate procedure)?5.31237Endoscopy, nose, surgical?6.30903–50Epistaxis (see Hemorrhage, Nasal), Hemorrhage, Control, Nasal, Complex?7.31576Laryngoscopy, fiberoptic ?8.31830Revision, tracheostomy scar ?9.30300Removal, foreign body, nose 10. 31575Laryngoscopy, fiberoptic, diagnostic Exercise 4.22 Cardiovascular System—Operative ReportsOperative Report #133207Pacemaker, heart, insertion; or Insertion, pacemaker, heartOperative Report #236590Removal, venous access deviceOperative Report #336556Central Venous Catheter Placement, insertion, central, non-tunneled Exercise 4.23 Cardiovascular System Review?1.33222Pacemaker, heart, relocate, skin pocket for pacemaker ?2.36870Arteriovenous Fistula, thrombectomy, graft; or Thrombectomy, arteriovenous fistula, graft ?3.33824Ductus Arteriosus, repair?4.37225-LT Revascularization, artery, femoral/popliteal ? 5.36582Venous Access Device, replacement?6.36833Arteriovenous Fistula, Revision, with thrombectomy ?7. 36215Catheterization, brachiocephalic artery?8. 33361Transcatheter Aortic Valve Replacement, femoral artery approach?9.33228Pacemaker, replacement, pulse generator 10.33517Coronary Artery Bypass Graft (CABG), arterial-venous bypass 33535Coronary Artery Bypass Graft (CABG), arterial bypassExercise 4.24 Digestive System—Endoscopy?1.43202Endoscopy, esophagus, biopsy 43217Endoscopy, esophagus, removal, polypNote: Modifier 59 or new subset modifiers would apply.?2.45315Proctosigmoidoscopy, removal of polyp?3.43270Esophagogastroduodenoscopy, transoral, ablation, polyp?4.43264Bile Duct, endoscopy, removal, calculus (stone)?5.45378-53Endoscopy, colon, exploration (the professional edition of CPT has a helpful Colonoscopy Decision Tree) Exercise 4.25 Digestive System—Operative ReportsOperative Report #143247Endoscopy, gastrointestinal, upper, foreign bodyOperative Report #245384Endoscopy, colon, removal, polypOperative Report #345330Sigmoidoscopy, explorationExercise 4.26 Digestive System—Hernia Repairs?1.49500Hernia, repair, inguinal, initial, child under 5 years?2.49656Hernia, repair, incisional, laparoscopic?3.49651Laparoscopy, hernia repair, inguinal, recurrent?4.49521Hernia, repair, inguinal, incarceratedNote: Mesh code is only coded with incisional and ventral hernia repairs.?5.49585Hernia, repair, umbilicus, reducible?6.49561 Hernia, repair, ventral, initial, strangulated49568Implantation, meshExercise 4.27 Digestive System—Operative ReportOperative Report #149505–LTHernia, repair, inguinal, child 5 years or older Note: Mesh code is only coded with incisional and ventral hernia repairs.Exercise 4.28 Digestive System Review?1.46930Hemorrhoids, destruction?2.43644Laparoscopy, gastric restrictive procedures?3.43245Esophagogastroduodenoscopy, transoral, dilation of gastric/duodenal stricture?4.47533 Bile Duct, drainage, catheter, placement?5.45388Colon, lesion, ablation?6.49322Laparoscopy, aspiration ?7.42809Removal, foreign body, pharynx ?8.47564Cholecystectomy, laparoscopic ?9.46611Anoscopy, removal, polyp10.42200PalatoplastyExercise 4.29 Urinary System—Cystoscopy?1.52332Cystourethroscopy, insertion, indwelling ureteral stent?2.52353Cystourethroscopy, lithotripsy ?3.52235Cystourethroscopy, with fulguration, lesion ?4.52282Cystourethroscopy, insertion, urethral stentExercise 4.30 Urinary System—Operative ReportsOperative Report #152352Cystourethroscopy, removal, calculus 52332–51–RTInsertion, stent, ureteral Note: Because the stent was inserted at the conclusion of the procedure, one can presume it is an indwelling ureteral stent.Operative Report #252234Cystourethroscopy, with fulguration, lesionExercise 4.31 Urinary System Review?1.51992Sling Operation, stress incontinence?2.50694 Ureter, stent, placement ?3.52235Cystourethroscopy, with fulguration, tumor?4.50920Fistula, closure, ureter?5.50200Kidney, biopsy?6.52352Cystourethroscopy, removal calculus ?7.52290Cystourethroscopy, with ureteral meatotomy (bilateral modifier not appropriate because the code description specifies “unilateral” or “bilateral”)?8.53265Urethra, lesion, destruction ?9.51701Insertion, catheter, urethral10.51525Cystotomy, excision, bladder diverticulumExercise 4.32 Male Genital System?1.54057Lesion, penis, destruction, laser surgery?2.10060Incision and drainage, abscess, skin?3.55845Prostatectomy, retropubic, radical?4. 55250Vasectomy?5. 54150Circumcision, surgical excision, neonateExercise 4.33 Male Genital System—Operative ReportsOperative Report #155875Prostate, brachytherapy, needle insertionOperative Report #254520–LTOrchiectomy, simpleExercise 4.34 Male Genital System Review?1.54840Spermatocele, excision?2.54056Penis, lesion, destruction, cryosurgery ?3.54690Orchiectomy, laparoscopic?4.55100Scrotum, abscess, incision and drainage ?5.54865Epididymis, exploration, biopsy?6.55866Laparoscopy, prostate, prostatectomy ?7.54240Penis, plethysmography?8.54406Penile Prosthesis, removal, inflatable ?9.54640–50Orchiopexy, inguinal approach10.54322Hypospadias, repair, one stage, meatal advancementExercise 4.35 Female Genital System?1.58670Laparoscopy, Ovary/Oviduct, Fulguration; Oviducts?2.49322Laparoscopy, aspiration, cyst?3.59812Abortion, incomplete?4.58120Dilation and curettage, corpus uteri?5.58545Laparoscopy, removal, leiomyomata?6.58150Hysterectomy, abdominal, totalExercise 4.36 Female Genital System—Operative ReportsOperative Report #157461LEEP Procedure, loop electrode conizationOperative Report #258558Hysteroscopy, surgical with biopsyExercise 4.37 Female Genital System Review? 1.57455Colposcopy, Cervix; or Colposcopy, Biopsy, Cervix? 2.56620Vulvectomy, simple, partial (use definitions at beginning of section) 3.58662Laparoscopy, destruction, lesion? 4. 58615Fallopian Tube, occlusion? 5.57023Incision and Drainage, hematoma, vagina ?6.56605Biopsy, vulva (external genitalia)57105Biopsy, vagina? 7.58554Hysterectomy, vaginal? 8.58561Hysteroscopy, removal, leiomyomata ? 9.58290Hysterectomy, vaginal 10.58356Ablation, endometrial, with ultrasound guidance ?Exercise 4.38 Endocrine System Review?1.60280Thyroglossal Duct, cyst, excision?2. 60500Parathyroid Gland, excision ?3.60260–50Thyroid Gland, excision, total, removal of all thyroid tissue?4.60650Laparoscopy, adrenal gland, excisionNote: There is a misleading index entry for this example. Adrenalectomy, Laparoscopic, leads to 50545, which is incorrect. ?5.60300Thyroid gland, cyst, aspiration Exercise 4.39 Nervous System?1.64445Injection, nerve, anesthetic?2.64702–F3Neuroplasty, digital nerve69990Operating microscope?3.62281Epidural, injection?4.64831–F5Neurorrhaphy64832–F8Exercise 4.40 Nervous System—Operative ReportOperative Report #1?63075Discectomy, cervicalExercise 4.41 Nervous System Review 1.62223Shunt, brain, creation 2.63704Myelomeningocele, repair 3.64782Excision, neuroma 4. 62311Epidural, injection 5.63272Laminectomy, for excision, intraspinal lesion, other than neoplasm 6.61312Craniotomy, surgery or Hematoma, brain, evacuation via craniotomy 7.63688Removal, pulse generator, spinal cord 8.64712Neuroplasty, peripheral nerve, sciatic 9.64408Nerves, injection, anesthetic10. 64840Suture, nerveExercise 4.42 Eye and Ocular Adnexa—Operative ReportOperative Report #166984–LTPhacoemulsification, removal, extracapsular cataractExercise 4.43 Eye and Ocular Adnexa Review?1.65222Removal, foreign body, cornea with slit lamp?2.67311Strabismus, repair, one horizontal muscle ? 3.68110Lesion, conjunctiva, excision?4.67413–RTOrbitotomy, with removal of foreign body ?5.67810–E1Biopsy, eyelid?6.67966Eyelid, repair, excisional ??7.67906–E1Blepharoptosis, repair, superior rectus technique with fascial sling?8.67825-E4Trichiasis, repair, epilation, by other than forceps?9.67914Ectropion, repair, suture10.67210Photocoagulation, lesion, retina Exercise 4.44 Auditory System—Operative ReportOperative Report #169436–50Tympanostomy, general anesthesia (–50 for bilateral)Exercise 4.45 Auditory System Review?1.69910Labyrinthectomy with mastoidectomy?2.69205–LTRemoval, foreign body, auditory canal, external, with anesthesia?3.69636Tympanoplasty, with antrotomy or mastoidotomy, with ossicular chain reconstruction?4.69930-50Implantation, cochlear device ?5.69005–LTHematoma, ear, external, incision and drainage (would expect to see documentation to explain why the procedure took an extensive amount of time)Chapter 4 Review: Coding for Facility?1.42305Incision and drainage, abscess, parotid gland?2.12044Repair, wound, intermediate ?3.57455Colposcopy, biopsy, cervix?4. 39401Mediastinoscopy, biopsy, mediastinal mass?5.19101–LTBiopsy, breast?6.69436–50Tympanostomy, general anesthesia ?7.26750–F5Fracture, phalanges, distal, closed treatment,?8.63005Laminectomy, for decompression, lumbar?9.36569 Catheterization, venous, central line see Central Venous Catheter Placement, Insertion, Peripheral without port or pump10.52235Cystourethroscopy, with fulguration, tumor 11.49507–50Hernia repair, inguinal, incarcerated12.11406Lesion, skin, excision, benign13.26010–FAFinger, abscess, incision and drainage26010–F114.31535Laryngoscopy, direct, biopsy15.69666Fistula, repair, oval window ear16.28298Bunion Repair, by phalanx osteotomy17.54060Lesion, penis, excision18.29834–LTArthroscopy, surgical, elbow19.59000Amniocentesis, diagnostic20. 58561Hysteroscopy, removal, leiomyomata 56740Bartholin’s Gland, excision Chapter 4 Review: Coding for Physician Services?1.61520–62 (physician #1)Cerebellopontine angle tumor, see brain, tumor, excision, cerebellopontine angle61520–62(physician # 2)?2.31255–50Endoscopy, sinuses, surgical31267-50 Instructional note under code 31256 states to code both 31255 and 31267 for this procedure?3.29821–LTArthroscopy, surgical, shoulder?4.27786–79Fracture, fibula, closed treatment?5.43274Cholangiopancreatography, Endoscopic Retrograde (ERCP), with placement, stent?6.59840Abortion, induced by dilation and curettage?7.68811–50Nasolacrimal duct, exploration, with anesthesia Note: Bilateral modifier applies because the code describes one duct.?8. 58661–22Laparoscopy, removal, fallopian tube?9. 44151Colectomy, total, open, with ileostomy10. 33208–54Pacemaker, heart, insertion (modifier for surgical care only)11.15783Dermabrasion12. 45378-53 Colonoscopy, flexible diagnosticChapter 5 Answer KeyChapter 5: RadiologyExercise 5.1 Diagnostic Radiology?1.74182MRI, see Magnetic resonance imaging, diagnostic, abdomen?2.74262Colonography, CT scan, diagnostic?3.72170X-ray, pelvis?4.72265 Myelography, spine, lumbosacral?5.74430Cystography, bladder?6. 72131CT scan without contrast, spine, lumbar ?7. 74177CT scan, with contrast, abdomen (Note: The note under code 74170 directs the coder to assign a code for a combined CT of abdomen and pelvis)8. 74241X-ray, gastrointestinal tractExercise 5.2 Diagnostic Ultrasound?1.76831Hysterosonography, see Ultrasound; Sonohysterography?2.76815Ultrasound, pregnant uterus?3.76641-50Ultrasound, breast?4.76770Ultrasound, kidney ?5.76870Ultrasound, scrotum Chapter 5 Review?1.73090–26X-ray, arm, lower Note: Modifier 26 is reported to identify the professional component of the procedure, which includes supervising the procedure, reading and interpreting the results, and documenting the interpretation in a report.?2.72142–26MRI, see magnetic resonance imaging (MRI), spine, cervical ?3.74430Cystography, bladderNote: Modifier 26 was not appended to the code because the description includes supervision and interpretation.?4.74178CT scan, without and with contrast, abdomen ?5.74280Barium enema?6.77761Radioelement, application, device placement, intracavitary ??7.70460–TCCT scan, with contrast, headNote: In this case, the radiology facility would report modifier TC to identify the technical component of the procedure, which includes performance of the actual procedure and expenses for supplies and equipment. ?8.76817Ultrasound, pregnant uterus ?9.76930Ultrasound, guidance, pericardiocentesis10.78104Bone Marrow, nuclear medicine, imaging11.77057Mammography, screening12.78265Gastrointestinal Exam, nuclear medicine, gastric empting imaging study, with small bowel transit13.77402Radiation therapy, treatment delivery, single area14.78707Kidney, Nuclear medicine, imaging15.73000X-ray, clavicle 16.78451 Myocardial, perfusion imaging17.72255Myelography, spine, thoracic18.76948Ultrasound, guidance, ova retrieval19.73560X-ray, knee20.76510 Ophthalmology, Diagnostic, ultrasound. Note that the range provided in the index (76511–76529) does not include the correct code 76510.21.76775Ultrasound, kidney (Code 76775, limited, is for a single organ.) (see CPT Assistant, May 1999.)22.73502–LTX-ray, hip 23.74247X-ray, gastrointestinal 24.71020X-ray, chest 25.70450CT scan, without contrast, headChapter 6 Answer KeyChapter 6: Pathology and Laboratory ServicesChapter 6 Review?1.88304Pathology and Laboratory, surgical pathology, gross and micro exam?2. 80178Therapeutic Drug Assay, lithium?3.82803Blood gases— pH, CO2, HCO3, pO2, pCO2?4.86688Antibody, HTLV-II ?5.85055Platelet assay?6.88025Autopsy, gross and micro exam?7.86510Histoplasmosis skin test?8.86280HAI Test See Hemagglutination Inhibition Test?9.80061Organ- or disease-oriented panel, lipid panel80051Organ- or disease-oriented panel, electrolyte10.83885Nickel 11.80402ACTH, see adrenocorticotropic hormone (ACTH), stimulation panel12.86695Herpes Simplex, antibody 13.85730Thromboplastin, partial time (PTT)85610Prothrombin time14.81003Urinalysis, automated 15.88331Pathology and Laboratory, surgical pathology, consultation, intraoperativeChapter 7 Answer KeyChapter 7: Evaluation and Management ServicesExercise 7.1 Evaluation and Management (History)1.HPI is brief (location, quality, and duration). Review of system(s) is problem specific. No PFSH documented. The history component of this visit would be expanded problem focused (brief HPI, problem-specific ROS, and expanded problem-focused PFSH including past, family, and social history). The history component is equal to the lowest category documented. 2.HPI is extended (location, severity, duration, and context). Review of systems is extended (two to nine systems). The PFSH is complete (two history areas documented). The history level is detailed; all three categories met this level.3.HPI is brief (location, severity, and duration). Review of system(s) is extended (two to nine systems), and PFSH is pertinent (medications). The history is determined by the lowest level from all three categories; therefore, the history level would be expanded problem focused.Exercise 7.2 Evaluation and Management (Physical Examination)prehensive examination: eight body systems were reviewed.2.Detailed examination. Note that the criteria for expanded problem focused and the detailed category is the same except in the level of specificity in the examination. Decisions on the level of specificity can be somewhat subjective.3.Expanded problem-focused examination: two systems reviewed (constitutional and integumentary).Exercise 7.3 Evaluation and Management (Medical Decision Making)1.Moderate complexity2. Low complexity3. StraightforwardExercise 7.4 Evaluation and Management Case StudyEstablished Patient HistoryMedical Decision MakingReview the code descriptions for 99211–99215. Need two out of the three key components. The case study revealed a detailed history and comprehensive examination, and the medical decision making was of moderate complexity. The correct E/M code selection is 99214.Exercise 7.5 Evaluation and Management?1. False ?2. True ?3. False?4. False?5. False?6. Pertinent—One area (past history)?7.Expanded problem focused—Systems reviewed: respiratory, integumentary, ENT (ears, nose, throat), cardiovascular, gastrointestinal. Expanded problem focused (two to nine systems)?8. Extended—Four elements: yesterday (duration); getting ready to go to church (context); vomited bile (quality); deafness at the start (associated signs/symptoms)?9.Five key elements (location, quality, duration, context, and modifying factors)10.Two body systems (respiratory and cardiovascular)Chapter 7 Review?1. 99211?2. 99203?3.99245?4.99202 ?5. 99291?6. 99281?7. 99396?8. 99213 ?9. 9921210. 9920311. 9923512. 99214Chapter 8 Answer KeyChapter 8: MedicineExercise 8.1 Immunizations1.90471Administration, immunization, one vaccine/toxoid90714Vaccines, tetanus and diphtheria toxoid 2.90460Administration, immunization, one vaccine/toxoid,90461 x 3 (each additional vaccine/toxoid component)90710Vaccines, Measles, Mumps, Rubella and Varicella 3.90471Administration, immunization, one vaccine/toxoid90472(add-on code) Immunization administration, each additional vaccine/toxoid90632Vaccines, hepatitis A90658Vaccines, influenza, for intramuscular use4.90470Administration, immunization, one vaccine/toxoid90651Vaccine, Human Papilloma Virus5.90471Administration, immunization, one vaccine/toxoid90748Vaccines, hepatitis B and Haemophilus influenzae BExercise 8.2 Psychiatry1.90885Psychiatric diagnosis, evaluation of records or reports2.90834Psychotherapy, Individual Patient/Family Member 3.90847Psychotherapy, family of patient Exercise 8.3 Dialysis1.90962Dialysis, end-stage renal disease see End-stage renal disease2.90969End-stage renal disease services90969(per day)3.90945Dialysis, peritoneal Exercise 8.4 Ophthalmology1.92018Gonioscopy (See note under code 92020)2.92004Ophthalmology, Diagnostic, Eye Exam, New patientExercise 8.5 Cardiovascular Services1.93025Electrocardiography, Rhythm, Microvolt T-wave Alternans2.92973-RCThrombectomy, percutaneous, coronary artery3.92977Thrombolysis, coronary vessels, infusion4.93015Stress Tests, cardiovascular5.92924–LCArtery, coronary, atherectomy6.92928–LDStent, placement, transcatheter, intracoronary 7.93040 ECG, see Electrocardiography. Electrocardiography, rhythm, tracing and evaluation8.93452Cardiac Catheterization, Left Heart, with VentriculographyExercise 8.6 Pulmonary Services1.94660CPAP, see Continuous Positive Airway Pressure 2.94060Spirometry 3.94450 Hypoxia, breathing responseExercise 8.7 Allergy and Clinical Immunology1.95017Allergy Tests, skin tests, venoms2.95120Allergen Immunotherapy, allergen extracts, injection and provisionExercise 8.8 Injections and Infusions1.96372Injection, intramuscular, therapeutic (Also, J3420 would be assigned for specific substance- B12.)2.96413Chemotherapy, intravenous, infusion96415(add-on code)Note: J code for Cisplatin would also be assigned.3.96360Infusion, Intravenous, Hydration 4.96420Chemotherapy, intra-arterial, pushExercise 8.9 Physical Medicine and Rehabilitation1.97032 x 2TENS, see Physical Medicine/Therapy/Occupational Therapy---Modalities, Electric stimulation, attended, manual2.97113 x 2Physical Medicine/Therapy/Occupational Therapy, aquatic therapy with exercises3.97602Wound, care, debridement, nonselective 4.97006Physical Medicine/Therapy/Occupational Therapy, athletic training, re-evaluation Chapter 8 Review1.90966End-stage renal disease services2.90471Administration, immunization, one vaccine/toxoid 90716 Vaccines, varicella3.93010EKG, see electrocardiography, evaluation4.94640Inhalation Treatment, Pressurized or Nonpressurized 5.93600Electrophysiology Procedure 6.96921Psoriasis Treatment7. 99502Home Services, newborn care8.96413Chemotherapy, intravenous, infusion96415 x 2 9.92924 Artery, atherectomy, coronary92925(additional branch)10.92556 Audiologic Function Tests, Audiometry, speech11.92928-LC Coronary Artery, angioplasty, with stent placement92973-RC Thrombectomy, percutaneous, coronary artery12. 99606Medication Therapy Management, pharmacist provided99607(additional 15 minutes)13.95863EMG, see electromyography, needle, extremities14.97761 x 2Prosthesis training 15.99050Special services, after-hours medical services Note: An E/M code would also be assigned in addition to this service.16.92014–25Ophthalmology, diagnostic, eye exam, established patient92283Opththalmology, diagnostic, color vision exam 17.98926Osteopathic Manipulation18.93925 Duplex scan, arterial studies, lower extremity 19.96372Injection, intramuscular, therapeutic99070Supply, materials Note: If this were a Medicare patient, the J code to identify the substance (steroid) would be reported 20.90832Psychotherapy, Individual Patient/Family MemberNote: CPT provides guidance to choose the code closest to the actual time.21.95807Sleep StudyChapter 9 Answer KeyChapter 9: AnesthesiaChapter 9 Review?1.00406–P2Anesthesia, breast?2.00948–P1AnesthesiaP1 Anesthesia, cervical cerclage ?3.00567–P4Anesthesia, heart, coronary artery bypass grafting?4. 01744–P1Anesthesia, arm, upper99100Anesthesia, special circumstances, extreme age?5.00702–P1Anesthesia, liver?6.00862-P2Anesthesia, nephrectomy?7.00580–P5Anesthesia, Heart, transplant99100 Anesthesia, special circumstances, extreme age99140 Anesthesia, special circumstances, emergency?8.00103-P1Anesthesia, Eyelid ?9. 00350–P3Anesthesia, neck99100 Anesthesia, special circumstances, extreme age 10.00540–P3Anesthesia, lungsChapter 10 Answer KeyChapter 10: HCPCS Level II Exercise 10.1 HCPCS Level II Codes1.A4611Battery, heavy duty; ventilator; or ventilator, battery2.H1010Education, family planning, nonmedical3.M0076Prolotherapy 4.J8520Capecitabine, oral, 150 mg 5.G0025Collagen, skin test6. L8000Mastectomy bra7. J9209Mesna8. E0188Sheepskin pad9. V2756 Eye case10. P2029 Congo red, bloodChapter 10 Review?1.E0186Mattress, air pressure?2.A4500Surgical, stocking ?3.J3420Vitamin, B12?4.A4357Drainage bag ?5.J1710Hydrocortisone-sodium phosphate?6.29540Strapping, ankleE0112Crutches ?7.51702Insertion, catheter, urethraA4355Catheter, irrigation supplies ?8.67938-E1Removal, foreign body, eyelid?9.11730–TAAvulsion, nails11732–T110.26055–F8Trigger finger repairChapter 11 Answer KeyChapter 11: Reimbursement in the Ambulatory Setting Quality Review Exercise1. Incorrect. The documentation supports a code for abdominal hysterectomy, not vaginal hysterectomy. Correct code should be 58150.2. Incorrect. The technique of hot biopsy forceps should be reflected in the code. Correct code 45384.3. Correct. The excised diameter is the size of the lesion plus the margins. 2.0 + 0.5 + 0.5 = 3.04. Incorrect. This procedure was performed laparoscopically; therefore, the correct code is 49651.5. Incorrect. Because all repairs were of the same type (simple) and same site classification, the lacerations may be added together. However, the code of 12053 is for intermediate wound repair, not simple. The correct CPT code is 12014. 6. Incorrect. The correct code should be 19083 for ultrasound guidance. Incorrect code 19085 describes MRI guidance.7. Incorrect. The benign tumor extended beyond the skin and would be assigned code 28041 for Excision, tumor, soft tissue. 8. Incorrect. The CPT code should be 27792 for distal fibula. The modifier should be LT for left.9. Incorrect. The CPT incorrectly identifies a ureteral stent. The correct code 52282 is for urethral stent.10. Incorrect. The patient has excess fluid; therefore, the physician performed a therapeutic amniocentesis (59001), not a diagnostic procedure.Answers to Additional Practice Exercises, Appendix B—2016Case Number Code(s) Index Entries 1.47563 Laparoscopy, biliary tract, cholecystectomy 2. 11441 Excision benign lesion, face; 1.0 cm11442 Excision benign lesion, face; 1.5 cm12051 Repair, intermediate face; 2.5 cm3.43235 Esophagogastroduodenoscopy, transoral, examination 43450–59 Dilation, esophagus 4.23650-RT Dislocation, shoulder, closed treatment with manipulation 5.66984-RT Cataract, removal/extraction, extracapsular 6. 64595 Removal, Neurostimulator7. 11642 Lesion, skin, excision, malignant 8.58671 Laparoscopy, ovary/oviduct, oviduct surgery 58120 Dilation and Curettage, corpus uteri 58301 Removal, intrauterine device (IUD)Note: Modifier –51 is for physician office use only. 9.52601 Prostate, excision, transurethral; or Prostatectomy, transurethral 10. 35556Bypass Graft, Femoral-Popliteal11.42415 Parotid gland, tumor, excision 12. 19120-RT Excision, breast, cyst 13.31541 Laryngoscopy, direct 42960 Hemorrhage, control, oropharynx Note: Modifier –51 could be used for the physician’s claim based on payer policy 14.43246 Esophagogastroduodenoscopy, transoral, placement, gastrostomy tube 15.35301-RT Thromboendarterectomy, carotid artery 16.59320 Cerclage, cervix, vaginal 17.27880-LT Amputation, leg, lower18.36830Arteriovenous Fistula, creation19.45378 Colonoscopy, proximal to splenic flexure 20.21556 Excision, tumor, neck 21.67901-E1 Eyelid, repair, blepharoptosis, see Blepharoptosis, ---Blepharoptosis, repair, frontalis muscle technique 22. 23120-RT Mumford Operation, see Claviculectomy, partial 23.43239 Esophagogastroduodenoscopy, transoral, biopsy 45331–59 Sigmoidoscopy, biopsy 24.43239 Esophagogastroduodenoscopy, Transoral, Biopsy 25.12013 Wound, repair, face, simple (sum of lengths) 26.29881-LT Arthroscopy, surgical, knee 27.11403 Excision, skin, lesion, benign 12032 Wound repair, arm, intermediate 28.55700 Prostate, biopsy; or Biopsy, prostate (procedure performed via ultrasound)52000-59 Cystourethroscopy (was not the approach for the biopsy)29.60220-RT Thyroidectomy, partial (note that hemithyroidectomy is removing only half of the thyroid- —one lobe)30.45330 Sigmoidoscopy, exploration 31.36558 Insertion, catheter, venous 32.49652 Hernia Repair, ventral, laparoscopic (note that in the CPT 2014 index, the entry for Hernia Repair, Ventral, Laparoscopic does not lead you to 49652 (range provided is 49654–49657). Further down the list is another entry that provides the correct code. 33.19081-LT Biopsy, breast, with localization device placement34. 11042 Debridement, skin, subcutaneous tissue35.64721-LT Carpal tunnel syndrome, decompression 36.54060 Lesion, penis, destruction, surgical excision 37. 62311 Injection, epidural, see Epidural injection 38.38525 Excision, lymphatic system, nodes 39. 59812 Abortion, incomplete40.65205-LT Removal, foreign body, eye, external eye 41.32608 Thoracoscopy, diagnostic with biopsy 31628 Bronchoscopy, biopsy 42. 42825 Tonsils, excision; or Excision, tonsils; or Tonsillectomy 43.11421 Lesion, skin, excision, benign 44.28515-T8 Fracture, phalanges, toe, with manipulation 28515-T9 Fracture, phalanges, toe, with manipulation45.23076-RT Tumor, shoulder, soft tissue, excision (lipoma extended beyond layers of skin) Note that these codes are not in numeric sequence 46. 45385 Colonoscopy, proximal to splenic flexure, with removal, polyp 47.26608-LT Fracture, metacarpal, percutaneous fixation 26608-59-LT 48.33207 Insertion, pacemaker, heart 49.68816-RT Nasolacrimal Duct, exploration with anesthesia 50.12032 Wound, repair, leg, intermediate 51.52204 Cystourethroscopy, biopsy. Fulguration was used for hemostasis, an integral part of the procedure. 52.36831 Thrombectomy, Dialysis Graft53.43235-59 Esophagogastroduodenoscopy, see Endoscopy, gastrointestinal, upper, exploration43450 Dilation, esophagus (In this case, a diagnostic EGD was performed, and after the scope was removed, the physician dilated the esophagus with a Maloney dilator. Since the dilation was performed outside of the scope, then combination code of 43249 would be incorrect. Maloney dilation would be coded 43450. (CPT Assistant, Spring 1994, addresses this subject) 54.11602 Excision, lesion, malignant (8 mm + 5mm + 5mm = 18 mm or 1.8 cm) 12032 Wound repair, arm, intermediate 55.31541 Laryngoscopy, Direct (with operating microscope or telescope) 56. 45380 Colonoscopy, proximal to splenic flexure, with biopsy (January 2004 CPT Assistant provided new advice about use of cold knife biopsy forceps. Removal of small polyps via cold knife is assigned CPT code 45380)57. 64421 Injection, Nerve Anesthetic 58.62311 Injection, Spinal cord 59.52281 Cystourethroscopy, Dilation, Urethra 60.20103-LT Wound, Exploration, penetrating trauma (The note before the 20100 codes lists the descriptions that are assigned to this range. The clues in the OP report stated that the stab wound was ENLARGED. Coagulation of small vessels was another.) 61.31622 Bronchoscopy, exploration 62.26055-F2 Trigger finger repair63.58558 Hysteroscopy, surgical with biopsy64.12013 Wound Repair, ears, simple 12042 Wound Repair, ears, intermediate 65.57421 Colposcopy, biopsy, cervix 66.59000 Amniocentesis, diagnostic67.58150 Hysterectomy, abdominal, total. Note: The National Correct Coding Initiative states that if an endoscopic procedure is converted to an open procedure, only the open procedure may be reported. Neither a surgical endoscopy nor a diagnostic endoscopy code should be reported with the open procedure code when an endoscopic procedure is converted to an open procedure. (Chapter 7 of NCCI Policy Manual for Medicare Services, Effective January 1, 2013).Robotic surgical procedures are reported using existing CPT codes (CPT Assistant, August 2012, page 13)68.19307-LT Mastectomy, modified radical; or Breast, excision, mastectomy Note: Modifier –58 may be applicable for the physician’s claim. 69. 50590-RT Lithotripsy, kidney; or Kidney, lithotripsy 70. 21356-RT Fracture, zygomatic arch, open treatment71.31526 Laryngoscopy, direct, diagnostic72.67800-E2 Chalazion, excision, single ................
................

In order to avoid copyright disputes, this page is only a partial summary.

Google Online Preview   Download